Allowed Publications
Slot System
Featured Buckets
Featured Buckets Admin
Reverse Chronological Sort
Allow Teaser Image

The top pediatric hospital medicine articles of 2020

Article Type
Changed
Wed, 12/08/2021 - 14:52

The year 2020 was unlike any in recent history, particularly for those working in health care. With the onset of the SARs-CoV-2 pandemic, many physicians were met with increasing clinical demands, and hospitalists served an instrumental role in providing medical care as the world faced an unprecedented need for health care resources.

Dr. Maura A. Steed is instructor of hospital medicine, Northwestern Memorial Hospital and Ann and Robert H. Lurie Children's Hospital of Chicago, Northwestern University School of Medicine, Chicago.
Dr. Maura A. Steed

In addition, 2020 was a year in which many of us reflected on inequities both inside and outside of medicine. Many in health care witnessed the disproportionate burden that the SARs-CoV-2 pandemic placed on communities of color and inequities pertaining to vaccine distribution.

Dr. Rachel Danielle Fisher, Hospice and Palliative Medicine, Michigan State University
Dr. Rachel Danielle Fisher

In spite of the challenges of 2020, the field of pediatric hospital medicine (PHM) has continued to grow and evolve, with an incredible amount of new literature published in 2020.

Dr. Nathan M. Money is a pediatric hospitalist at Primary Children's Hospital, University of Utah School of Medicine, Salt Lake City.
Dr. Nathan M. Money

In this article, we identify the top 10 articles published in 2020, 5 of which are summarized below. These articles were presented at the Pediatric Update at SHM Converge 2021.
 

The top 5 articles

Association between parent comfort with English and adverse events among hospitalized children

Khan A et al. JAMA Pediatrics. December 2020.1

Background: Hospitalized children experience similar rates of medical errors compared to adult patients, but higher rates in areas that could cause harm.1 A major contributor to medical errors is communication failure, which language barriers frequently contribute to. Single-center data suggest that pediatric patients of families with limited comfort with English experience increased adverse events,2 but multicenter data are lacking.

Findings: This prospective cohort study observed adverse event rates among 2,148 patients from seven teaching hospitals from December 2014 to January 2017. Survey data revealed 147 of 1,666 (9%) parents of patient families expressed limited comfort in English, and Spanish was the predominant language in this group (71%). There were 217 adverse events reported, 142 (65%) of which were deemed preventable by study personnel. Nearly twice as many children of parents with limited comfort with English experienced an adverse event when compared to their English-speaking counterparts (26 of 147 [17.7%] vs. 146 of 1,519 [9.6%]; adjusted odds ratio, 2.1; 95% confidence interval, 1.2-3.7). Interpreter use was not measured.

Impact to practice: Children of parents with limited comfort with English are nearly twice as likely to experience adverse events when hospitalized. Hospitals should reflect on current practice and make efforts to improve their ability to identify and communicate with this vulnerable cohort.
 

Saline-lock versus continuous infusion: Maintaining peripheral intravenous catheter access in children

Yeung F et al. Hospital Pediatrics. December 2020.3

Background: Peripheral intravenous catheter (PIV) insertion is performed on most hospitalized children. Unfortunately, PIVs frequently fail and need to be replaced. There is a widespread perception that infusing a crystalloid solution at a low rate through a PIV, a strategy known as “to keep vein open” (TKO) prolongs the patency of PIVs, however there is a lack of evidence to support this practice.4Findings: In this prospective, time-allocated study, 172 children were allocated to either a TKO strategy or a saline-lock strategy with a primary outcome of duration of PIV patency.3 Secondary outcomes included PIV–related complication rates and patient and caregiver satisfaction. The mean duration of PIV patency was 41.68 hours in the TKO group and 44.05 hours in the saline-lock group, which did not meet the prespecified definition of a clinically significant difference. There was no significant difference in prevalence of PIV-associated complications and patient satisfaction was similar between the two groups.

Impact to practice: Running fluid “to keep vein open” does not increase the duration of PIV patency compared to intermittent saline locks. Given that a TKO strategy limits a patient’s mobility, this low-value practice can be discontinued without increasing the risk of PIV failure.
 

 

 

Intensive care unit utilization after adoption of a ward-based high flow nasal cannula protocol

Coon ER et al. Journal of Hospital Medicine. June 2020.5



Background: High Flow Nasal Cannula (HFNC) has been widely adopted for escalation of respiratory support in patients with bronchiolitis; however, its use is dictated by highly variant local protocols.6 Small-scale randomized control trials and systematic reviews show that early HFNC initiation in mild to moderate disease does not change patient outcomes.7Findings: In this retrospective cohort study of ward-based HFNC, the authors used the Pediatric Health Information System database to identify 12 hospitals that had adopted ward-based HFNC protocols. The study used an interrupted time series analysis to compare outcomes for patients ages 3-24 months hospitalized with bronchiolitis (n = 32,809) in the three seasons before and after protocol adoption. Ward-based HFNC adoption paradoxically increased ICU admission (absolute increase 3.1%, 95% confidence interval, 2.8-3.4%) and ICU length of stay (absolute difference 9.1 days/100 patients, 95% CI, 5.1-13.2). Total length of stay and rates of mechanical ventilation were similar between groups.5Impact to practice: Ward-based HFNC protocols are associated with increased ICU utilization. As bronchiolitis is the leading diagnosis in pediatrics, pediatric hospitals can lead ward-based quality efforts to decrease HFNC overutilization focused on decreased initiation or deimplementation.
 

Lower versus traditional treatment threshold for neonatal hypoglycemia

Van Kempen AAMW et al. New England Journal of Medicine. February 2020.8

Background: Hypoglycemia is the most common metabolic abnormality in newborns, and up to 30% of newborns are routinely monitored for hypoglycemia. There is no consensus regarding the appropriate threshold at which hypoglycemia should be treated in order to prevent neurologic injury. Prior studies of neonatal hypoglycemia have largely been observation and have yielded conflicting results.8Findings: In this multicenter, randomized, noninferiority trial, 689 infants born at 35 weeks gestational age or later with risk factors for hypoglycemia and a measured blood glucose of 36-46 mg/dL were randomized to either a lower glucose treatment threshold (36 mg/dL) or traditional glucose treatment threshold (47 mg/dL). The primary outcome was psychomotor development at 18 months, assessed via the Bayley Scale of Infant and Toddler Development, third edition. There was no significant difference in cognitive or motor scores at 18 months. The lower treatment threshold group had a higher frequency of severe hypoglycemia (< 36 mg/dL) and were more likely to have four or more episodes of hypoglycemia. The traditional treatment threshold group had more supplemental feeding and more IV glucose administration. Length of stay for the mother and baby did not differ between groups.8

Impact to practice: This prospective, randomized study suggests that reducing the treatment threshold for neonatal hypoglycemia did not affect neurodevelopmental at 18 months of age. In contrast, a recent meta-analysis by Shah et al. suggested that neonatal hypoglycemia was not associated with adverse neurodevelopmental outcomes in early childhood; however, differences in rates of neurodevelopmental impairment, low literacy, and low numeracy were detectable by age five.9
 

Factors associated with family experience in pediatric inpatient care

Feng JY et al. Pediatrics. March 2020 Mar.10

Background: Positive patient experience is associated with better health care outcomes and reduced health care use.11 Consequently, patient experience surveys have played a larger role in public reporting, financial risk sharing arrangements, and pay for performance programs. While adult studies have examined the importance of specific care dimensions for patient experience, data are lacking for inpatient pediatric populations.

Findings: A retrospective study collected Hospital Consumer Assessment of Healthcare Providers and Systems (HCAHPS) surveys from 17,727 patients in 69 hospitals within the United States over a 14-month period.10 Of the 10 care dimensions analyzed, child comfort (aOR 1.50; 95% CI, 1.41-1.60) and nurse-parent communication (aOR 1.50; 95% CI, 1.42-1.58) were most strongly associated with a family’s willingness to recommend a hospital. Additional associated indices included preparing to leave the hospital (aOR 1.34; 95% CI, 1.27-1.41), doctor-parent communication (aOR 1.28; 95% CI, 1.21–1.35), and keeping parents informed (aOR 1.25; 95% CI, 1.18-1.33). Privacy and quietness, which are associated with positive patient experience in adult studies, were not significantly associated with willingness to recommend in this cohort.

Impact to practice: Hospitals seeking to improve patient experience will benefit most by focusing on improving patient comfort and nurse-parent communication. Factors that increase adult patient satisfaction may not be as important to the pediatric population and their families.



The other five articles that comprised the top 10 are listed below:
 

Comparison of as-needed and scheduled posthospitalization follow-up for children hospitalized for bronchiolitis

Coon ER et al. JAMA Pediatrics. September 2020.12

Clinical prediction rule for distinguishing bacterial from aseptic meningitis

Mintegi S et al. Pediatrics. September 2020.13

The Michigan Appropriateness Guide for Intravenous Catheters in Pediatrics: miniMAGIC Ullman AJ et al. Pediatrics. June 2020.14

A structured neonatal parenting elective: An approach for parenting leave during residency

Cree-Green M et al. Academic Pediatrics. Aug 2020.15

The KidzMed project: Teaching children to swallow tablet medication

Tse Y et al. Archives of Disease in Childhood. November 2020.16

Dr. Steed is an internal medicine and pediatrics hospitalist at Northwestern Memorial Hospital and Ann and Robert H. Lurie’s Children’s Hospital of Chicago. Dr. Fisher is a current fellow in hospice and palliative medicine and a clinical assistant professor at Michigan State University. Dr. Money is an assistant professor of pediatrics at the University of Utah and a fellowship-trained pediatric hospitalist at Utah Valley Hospital and Primary Children’s Hospital.

References

1. Khan A et al. Association between parent comfort with english and adverse events among hospitalized children. JAMA Pediatr. 2020 Dec 1;174(12):e203215. doi: 10.1001/jamapediatrics.2020.3215.

2. Wasserman M et al. Identifying and preventing medical errors in patients with limited English proficiency: Key findings and tools for the field. J Healthc Qual. May-Jun 2014;36(3):5-16. doi: 10.1111/jhq.12065.

3. Yeung F et al. Saline-lock versus continuous infusion: Maintaining peripheral intravenous catheter access in children. Hosp Pediatr. 2020 Dec;10(12):1038-43. doi: 10.1542/hpeds.2020-0137.

4. Mok E et al. A randomized controlled trial for maintaining peripheral intravenous lock in children. Int J Nurs Pract. 2007 Feb;13(1):33-45. doi: 10.1111/j.1440-172X.2006.00607.x.

5. Coon ER et al. Intensive care unit utilization after adoption of a ward-based high-flow nasal cannula protocol. J Hosp Med. 2020 Jun;15(6):325-30. doi: 10.12788/jhm.3417.

6. Kalburgi S and Halley T. High-flow nasal cannula use outside of the ICU setting. Pediatrics. 2020;146(5):e20194083. doi: 10.1542/peds.2019-4083.

7. Leyenaar JK and Ralston SL. Widespread adoption of low-value therapy: The case of bronchiolitis and high-flow oxygen. Pediatrics. 2020 Nov;146(5):e2020021188. doi: 10.1542/peds.2020-021188.

8. Van Kempen AAMW et al. Lower versus traditional treatment threshold for neonatal hypoglycemia. N Engl J Med. 2020 Feb 6;382(6):534-44. doi: 10.1056/NEJMoa1905593.

9. Shah R et al. Neonatal glycaemia and neurodevelopmental outcomes: A systematic review and meta-analysis. Neonatology. 2019;115(2):116-26. doi: 10.1159/000492859.

10. Feng JY et al. Factors associated with family experience in pediatric inpatient care. Pediatrics. 2020 Mar;145(3):e20191264. doi: 10.1542/peds.2019-1264.

11. Anhang Price R et al. Examining the role of patient experience surveys in measuring health care quality. Med Care Res Rev. 2014 Oct;71(5):522-54. doi: 10.1177/1077558714541480.

12. Coon ER et al. Comparison of as-needed and scheduled posthospitalization follow-up for children hospitalized for bronchiolitis: The Bronchiolitis Follow-up Intervention Trial (BeneFIT) randomized clinical trial. JAMA Pediatr. 2020 Sep 1;174(9):e201937. doi: 10.1001/jamapediatrics.2020.1937.

13. Mintegi S et al. Clinical prediction rule for distinguishing bacterial from aseptic meningitis. Pediatrics. 2020 Sept;146(3): e20201126. doi: 10.1542/peds.2020-1126.

14. Ullman AJ et al. The Michigan Appropriateness Guide for Intravenous Catheters in pediatrics: miniMAGIC. Pediatrics. 2020 Jun;145(Suppl 3):S269-S284. doi: 10.1542/peds.2019-3474I.

15. Cree-Green M et al. A structured neonatal parenting elective: an approach for parenting leave during residency. Acad Pediatr. 2021 Jan-Feb;21(1):16-18. doi: 10.1016/j.acap.2020.02.008.

16. Tse Y et al. The KidzMed project: Teaching children to swallow tablet medication. Arch Dis Child. 2020 Nov;105(11):1105-7. doi: 10.1136/archdischild-2019-317512.

Publications
Topics
Sections

The year 2020 was unlike any in recent history, particularly for those working in health care. With the onset of the SARs-CoV-2 pandemic, many physicians were met with increasing clinical demands, and hospitalists served an instrumental role in providing medical care as the world faced an unprecedented need for health care resources.

Dr. Maura A. Steed is instructor of hospital medicine, Northwestern Memorial Hospital and Ann and Robert H. Lurie Children's Hospital of Chicago, Northwestern University School of Medicine, Chicago.
Dr. Maura A. Steed

In addition, 2020 was a year in which many of us reflected on inequities both inside and outside of medicine. Many in health care witnessed the disproportionate burden that the SARs-CoV-2 pandemic placed on communities of color and inequities pertaining to vaccine distribution.

Dr. Rachel Danielle Fisher, Hospice and Palliative Medicine, Michigan State University
Dr. Rachel Danielle Fisher

In spite of the challenges of 2020, the field of pediatric hospital medicine (PHM) has continued to grow and evolve, with an incredible amount of new literature published in 2020.

Dr. Nathan M. Money is a pediatric hospitalist at Primary Children's Hospital, University of Utah School of Medicine, Salt Lake City.
Dr. Nathan M. Money

In this article, we identify the top 10 articles published in 2020, 5 of which are summarized below. These articles were presented at the Pediatric Update at SHM Converge 2021.
 

The top 5 articles

Association between parent comfort with English and adverse events among hospitalized children

Khan A et al. JAMA Pediatrics. December 2020.1

Background: Hospitalized children experience similar rates of medical errors compared to adult patients, but higher rates in areas that could cause harm.1 A major contributor to medical errors is communication failure, which language barriers frequently contribute to. Single-center data suggest that pediatric patients of families with limited comfort with English experience increased adverse events,2 but multicenter data are lacking.

Findings: This prospective cohort study observed adverse event rates among 2,148 patients from seven teaching hospitals from December 2014 to January 2017. Survey data revealed 147 of 1,666 (9%) parents of patient families expressed limited comfort in English, and Spanish was the predominant language in this group (71%). There were 217 adverse events reported, 142 (65%) of which were deemed preventable by study personnel. Nearly twice as many children of parents with limited comfort with English experienced an adverse event when compared to their English-speaking counterparts (26 of 147 [17.7%] vs. 146 of 1,519 [9.6%]; adjusted odds ratio, 2.1; 95% confidence interval, 1.2-3.7). Interpreter use was not measured.

Impact to practice: Children of parents with limited comfort with English are nearly twice as likely to experience adverse events when hospitalized. Hospitals should reflect on current practice and make efforts to improve their ability to identify and communicate with this vulnerable cohort.
 

Saline-lock versus continuous infusion: Maintaining peripheral intravenous catheter access in children

Yeung F et al. Hospital Pediatrics. December 2020.3

Background: Peripheral intravenous catheter (PIV) insertion is performed on most hospitalized children. Unfortunately, PIVs frequently fail and need to be replaced. There is a widespread perception that infusing a crystalloid solution at a low rate through a PIV, a strategy known as “to keep vein open” (TKO) prolongs the patency of PIVs, however there is a lack of evidence to support this practice.4Findings: In this prospective, time-allocated study, 172 children were allocated to either a TKO strategy or a saline-lock strategy with a primary outcome of duration of PIV patency.3 Secondary outcomes included PIV–related complication rates and patient and caregiver satisfaction. The mean duration of PIV patency was 41.68 hours in the TKO group and 44.05 hours in the saline-lock group, which did not meet the prespecified definition of a clinically significant difference. There was no significant difference in prevalence of PIV-associated complications and patient satisfaction was similar between the two groups.

Impact to practice: Running fluid “to keep vein open” does not increase the duration of PIV patency compared to intermittent saline locks. Given that a TKO strategy limits a patient’s mobility, this low-value practice can be discontinued without increasing the risk of PIV failure.
 

 

 

Intensive care unit utilization after adoption of a ward-based high flow nasal cannula protocol

Coon ER et al. Journal of Hospital Medicine. June 2020.5



Background: High Flow Nasal Cannula (HFNC) has been widely adopted for escalation of respiratory support in patients with bronchiolitis; however, its use is dictated by highly variant local protocols.6 Small-scale randomized control trials and systematic reviews show that early HFNC initiation in mild to moderate disease does not change patient outcomes.7Findings: In this retrospective cohort study of ward-based HFNC, the authors used the Pediatric Health Information System database to identify 12 hospitals that had adopted ward-based HFNC protocols. The study used an interrupted time series analysis to compare outcomes for patients ages 3-24 months hospitalized with bronchiolitis (n = 32,809) in the three seasons before and after protocol adoption. Ward-based HFNC adoption paradoxically increased ICU admission (absolute increase 3.1%, 95% confidence interval, 2.8-3.4%) and ICU length of stay (absolute difference 9.1 days/100 patients, 95% CI, 5.1-13.2). Total length of stay and rates of mechanical ventilation were similar between groups.5Impact to practice: Ward-based HFNC protocols are associated with increased ICU utilization. As bronchiolitis is the leading diagnosis in pediatrics, pediatric hospitals can lead ward-based quality efforts to decrease HFNC overutilization focused on decreased initiation or deimplementation.
 

Lower versus traditional treatment threshold for neonatal hypoglycemia

Van Kempen AAMW et al. New England Journal of Medicine. February 2020.8

Background: Hypoglycemia is the most common metabolic abnormality in newborns, and up to 30% of newborns are routinely monitored for hypoglycemia. There is no consensus regarding the appropriate threshold at which hypoglycemia should be treated in order to prevent neurologic injury. Prior studies of neonatal hypoglycemia have largely been observation and have yielded conflicting results.8Findings: In this multicenter, randomized, noninferiority trial, 689 infants born at 35 weeks gestational age or later with risk factors for hypoglycemia and a measured blood glucose of 36-46 mg/dL were randomized to either a lower glucose treatment threshold (36 mg/dL) or traditional glucose treatment threshold (47 mg/dL). The primary outcome was psychomotor development at 18 months, assessed via the Bayley Scale of Infant and Toddler Development, third edition. There was no significant difference in cognitive or motor scores at 18 months. The lower treatment threshold group had a higher frequency of severe hypoglycemia (< 36 mg/dL) and were more likely to have four or more episodes of hypoglycemia. The traditional treatment threshold group had more supplemental feeding and more IV glucose administration. Length of stay for the mother and baby did not differ between groups.8

Impact to practice: This prospective, randomized study suggests that reducing the treatment threshold for neonatal hypoglycemia did not affect neurodevelopmental at 18 months of age. In contrast, a recent meta-analysis by Shah et al. suggested that neonatal hypoglycemia was not associated with adverse neurodevelopmental outcomes in early childhood; however, differences in rates of neurodevelopmental impairment, low literacy, and low numeracy were detectable by age five.9
 

Factors associated with family experience in pediatric inpatient care

Feng JY et al. Pediatrics. March 2020 Mar.10

Background: Positive patient experience is associated with better health care outcomes and reduced health care use.11 Consequently, patient experience surveys have played a larger role in public reporting, financial risk sharing arrangements, and pay for performance programs. While adult studies have examined the importance of specific care dimensions for patient experience, data are lacking for inpatient pediatric populations.

Findings: A retrospective study collected Hospital Consumer Assessment of Healthcare Providers and Systems (HCAHPS) surveys from 17,727 patients in 69 hospitals within the United States over a 14-month period.10 Of the 10 care dimensions analyzed, child comfort (aOR 1.50; 95% CI, 1.41-1.60) and nurse-parent communication (aOR 1.50; 95% CI, 1.42-1.58) were most strongly associated with a family’s willingness to recommend a hospital. Additional associated indices included preparing to leave the hospital (aOR 1.34; 95% CI, 1.27-1.41), doctor-parent communication (aOR 1.28; 95% CI, 1.21–1.35), and keeping parents informed (aOR 1.25; 95% CI, 1.18-1.33). Privacy and quietness, which are associated with positive patient experience in adult studies, were not significantly associated with willingness to recommend in this cohort.

Impact to practice: Hospitals seeking to improve patient experience will benefit most by focusing on improving patient comfort and nurse-parent communication. Factors that increase adult patient satisfaction may not be as important to the pediatric population and their families.



The other five articles that comprised the top 10 are listed below:
 

Comparison of as-needed and scheduled posthospitalization follow-up for children hospitalized for bronchiolitis

Coon ER et al. JAMA Pediatrics. September 2020.12

Clinical prediction rule for distinguishing bacterial from aseptic meningitis

Mintegi S et al. Pediatrics. September 2020.13

The Michigan Appropriateness Guide for Intravenous Catheters in Pediatrics: miniMAGIC Ullman AJ et al. Pediatrics. June 2020.14

A structured neonatal parenting elective: An approach for parenting leave during residency

Cree-Green M et al. Academic Pediatrics. Aug 2020.15

The KidzMed project: Teaching children to swallow tablet medication

Tse Y et al. Archives of Disease in Childhood. November 2020.16

Dr. Steed is an internal medicine and pediatrics hospitalist at Northwestern Memorial Hospital and Ann and Robert H. Lurie’s Children’s Hospital of Chicago. Dr. Fisher is a current fellow in hospice and palliative medicine and a clinical assistant professor at Michigan State University. Dr. Money is an assistant professor of pediatrics at the University of Utah and a fellowship-trained pediatric hospitalist at Utah Valley Hospital and Primary Children’s Hospital.

References

1. Khan A et al. Association between parent comfort with english and adverse events among hospitalized children. JAMA Pediatr. 2020 Dec 1;174(12):e203215. doi: 10.1001/jamapediatrics.2020.3215.

2. Wasserman M et al. Identifying and preventing medical errors in patients with limited English proficiency: Key findings and tools for the field. J Healthc Qual. May-Jun 2014;36(3):5-16. doi: 10.1111/jhq.12065.

3. Yeung F et al. Saline-lock versus continuous infusion: Maintaining peripheral intravenous catheter access in children. Hosp Pediatr. 2020 Dec;10(12):1038-43. doi: 10.1542/hpeds.2020-0137.

4. Mok E et al. A randomized controlled trial for maintaining peripheral intravenous lock in children. Int J Nurs Pract. 2007 Feb;13(1):33-45. doi: 10.1111/j.1440-172X.2006.00607.x.

5. Coon ER et al. Intensive care unit utilization after adoption of a ward-based high-flow nasal cannula protocol. J Hosp Med. 2020 Jun;15(6):325-30. doi: 10.12788/jhm.3417.

6. Kalburgi S and Halley T. High-flow nasal cannula use outside of the ICU setting. Pediatrics. 2020;146(5):e20194083. doi: 10.1542/peds.2019-4083.

7. Leyenaar JK and Ralston SL. Widespread adoption of low-value therapy: The case of bronchiolitis and high-flow oxygen. Pediatrics. 2020 Nov;146(5):e2020021188. doi: 10.1542/peds.2020-021188.

8. Van Kempen AAMW et al. Lower versus traditional treatment threshold for neonatal hypoglycemia. N Engl J Med. 2020 Feb 6;382(6):534-44. doi: 10.1056/NEJMoa1905593.

9. Shah R et al. Neonatal glycaemia and neurodevelopmental outcomes: A systematic review and meta-analysis. Neonatology. 2019;115(2):116-26. doi: 10.1159/000492859.

10. Feng JY et al. Factors associated with family experience in pediatric inpatient care. Pediatrics. 2020 Mar;145(3):e20191264. doi: 10.1542/peds.2019-1264.

11. Anhang Price R et al. Examining the role of patient experience surveys in measuring health care quality. Med Care Res Rev. 2014 Oct;71(5):522-54. doi: 10.1177/1077558714541480.

12. Coon ER et al. Comparison of as-needed and scheduled posthospitalization follow-up for children hospitalized for bronchiolitis: The Bronchiolitis Follow-up Intervention Trial (BeneFIT) randomized clinical trial. JAMA Pediatr. 2020 Sep 1;174(9):e201937. doi: 10.1001/jamapediatrics.2020.1937.

13. Mintegi S et al. Clinical prediction rule for distinguishing bacterial from aseptic meningitis. Pediatrics. 2020 Sept;146(3): e20201126. doi: 10.1542/peds.2020-1126.

14. Ullman AJ et al. The Michigan Appropriateness Guide for Intravenous Catheters in pediatrics: miniMAGIC. Pediatrics. 2020 Jun;145(Suppl 3):S269-S284. doi: 10.1542/peds.2019-3474I.

15. Cree-Green M et al. A structured neonatal parenting elective: an approach for parenting leave during residency. Acad Pediatr. 2021 Jan-Feb;21(1):16-18. doi: 10.1016/j.acap.2020.02.008.

16. Tse Y et al. The KidzMed project: Teaching children to swallow tablet medication. Arch Dis Child. 2020 Nov;105(11):1105-7. doi: 10.1136/archdischild-2019-317512.

The year 2020 was unlike any in recent history, particularly for those working in health care. With the onset of the SARs-CoV-2 pandemic, many physicians were met with increasing clinical demands, and hospitalists served an instrumental role in providing medical care as the world faced an unprecedented need for health care resources.

Dr. Maura A. Steed is instructor of hospital medicine, Northwestern Memorial Hospital and Ann and Robert H. Lurie Children's Hospital of Chicago, Northwestern University School of Medicine, Chicago.
Dr. Maura A. Steed

In addition, 2020 was a year in which many of us reflected on inequities both inside and outside of medicine. Many in health care witnessed the disproportionate burden that the SARs-CoV-2 pandemic placed on communities of color and inequities pertaining to vaccine distribution.

Dr. Rachel Danielle Fisher, Hospice and Palliative Medicine, Michigan State University
Dr. Rachel Danielle Fisher

In spite of the challenges of 2020, the field of pediatric hospital medicine (PHM) has continued to grow and evolve, with an incredible amount of new literature published in 2020.

Dr. Nathan M. Money is a pediatric hospitalist at Primary Children's Hospital, University of Utah School of Medicine, Salt Lake City.
Dr. Nathan M. Money

In this article, we identify the top 10 articles published in 2020, 5 of which are summarized below. These articles were presented at the Pediatric Update at SHM Converge 2021.
 

The top 5 articles

Association between parent comfort with English and adverse events among hospitalized children

Khan A et al. JAMA Pediatrics. December 2020.1

Background: Hospitalized children experience similar rates of medical errors compared to adult patients, but higher rates in areas that could cause harm.1 A major contributor to medical errors is communication failure, which language barriers frequently contribute to. Single-center data suggest that pediatric patients of families with limited comfort with English experience increased adverse events,2 but multicenter data are lacking.

Findings: This prospective cohort study observed adverse event rates among 2,148 patients from seven teaching hospitals from December 2014 to January 2017. Survey data revealed 147 of 1,666 (9%) parents of patient families expressed limited comfort in English, and Spanish was the predominant language in this group (71%). There were 217 adverse events reported, 142 (65%) of which were deemed preventable by study personnel. Nearly twice as many children of parents with limited comfort with English experienced an adverse event when compared to their English-speaking counterparts (26 of 147 [17.7%] vs. 146 of 1,519 [9.6%]; adjusted odds ratio, 2.1; 95% confidence interval, 1.2-3.7). Interpreter use was not measured.

Impact to practice: Children of parents with limited comfort with English are nearly twice as likely to experience adverse events when hospitalized. Hospitals should reflect on current practice and make efforts to improve their ability to identify and communicate with this vulnerable cohort.
 

Saline-lock versus continuous infusion: Maintaining peripheral intravenous catheter access in children

Yeung F et al. Hospital Pediatrics. December 2020.3

Background: Peripheral intravenous catheter (PIV) insertion is performed on most hospitalized children. Unfortunately, PIVs frequently fail and need to be replaced. There is a widespread perception that infusing a crystalloid solution at a low rate through a PIV, a strategy known as “to keep vein open” (TKO) prolongs the patency of PIVs, however there is a lack of evidence to support this practice.4Findings: In this prospective, time-allocated study, 172 children were allocated to either a TKO strategy or a saline-lock strategy with a primary outcome of duration of PIV patency.3 Secondary outcomes included PIV–related complication rates and patient and caregiver satisfaction. The mean duration of PIV patency was 41.68 hours in the TKO group and 44.05 hours in the saline-lock group, which did not meet the prespecified definition of a clinically significant difference. There was no significant difference in prevalence of PIV-associated complications and patient satisfaction was similar between the two groups.

Impact to practice: Running fluid “to keep vein open” does not increase the duration of PIV patency compared to intermittent saline locks. Given that a TKO strategy limits a patient’s mobility, this low-value practice can be discontinued without increasing the risk of PIV failure.
 

 

 

Intensive care unit utilization after adoption of a ward-based high flow nasal cannula protocol

Coon ER et al. Journal of Hospital Medicine. June 2020.5



Background: High Flow Nasal Cannula (HFNC) has been widely adopted for escalation of respiratory support in patients with bronchiolitis; however, its use is dictated by highly variant local protocols.6 Small-scale randomized control trials and systematic reviews show that early HFNC initiation in mild to moderate disease does not change patient outcomes.7Findings: In this retrospective cohort study of ward-based HFNC, the authors used the Pediatric Health Information System database to identify 12 hospitals that had adopted ward-based HFNC protocols. The study used an interrupted time series analysis to compare outcomes for patients ages 3-24 months hospitalized with bronchiolitis (n = 32,809) in the three seasons before and after protocol adoption. Ward-based HFNC adoption paradoxically increased ICU admission (absolute increase 3.1%, 95% confidence interval, 2.8-3.4%) and ICU length of stay (absolute difference 9.1 days/100 patients, 95% CI, 5.1-13.2). Total length of stay and rates of mechanical ventilation were similar between groups.5Impact to practice: Ward-based HFNC protocols are associated with increased ICU utilization. As bronchiolitis is the leading diagnosis in pediatrics, pediatric hospitals can lead ward-based quality efforts to decrease HFNC overutilization focused on decreased initiation or deimplementation.
 

Lower versus traditional treatment threshold for neonatal hypoglycemia

Van Kempen AAMW et al. New England Journal of Medicine. February 2020.8

Background: Hypoglycemia is the most common metabolic abnormality in newborns, and up to 30% of newborns are routinely monitored for hypoglycemia. There is no consensus regarding the appropriate threshold at which hypoglycemia should be treated in order to prevent neurologic injury. Prior studies of neonatal hypoglycemia have largely been observation and have yielded conflicting results.8Findings: In this multicenter, randomized, noninferiority trial, 689 infants born at 35 weeks gestational age or later with risk factors for hypoglycemia and a measured blood glucose of 36-46 mg/dL were randomized to either a lower glucose treatment threshold (36 mg/dL) or traditional glucose treatment threshold (47 mg/dL). The primary outcome was psychomotor development at 18 months, assessed via the Bayley Scale of Infant and Toddler Development, third edition. There was no significant difference in cognitive or motor scores at 18 months. The lower treatment threshold group had a higher frequency of severe hypoglycemia (< 36 mg/dL) and were more likely to have four or more episodes of hypoglycemia. The traditional treatment threshold group had more supplemental feeding and more IV glucose administration. Length of stay for the mother and baby did not differ between groups.8

Impact to practice: This prospective, randomized study suggests that reducing the treatment threshold for neonatal hypoglycemia did not affect neurodevelopmental at 18 months of age. In contrast, a recent meta-analysis by Shah et al. suggested that neonatal hypoglycemia was not associated with adverse neurodevelopmental outcomes in early childhood; however, differences in rates of neurodevelopmental impairment, low literacy, and low numeracy were detectable by age five.9
 

Factors associated with family experience in pediatric inpatient care

Feng JY et al. Pediatrics. March 2020 Mar.10

Background: Positive patient experience is associated with better health care outcomes and reduced health care use.11 Consequently, patient experience surveys have played a larger role in public reporting, financial risk sharing arrangements, and pay for performance programs. While adult studies have examined the importance of specific care dimensions for patient experience, data are lacking for inpatient pediatric populations.

Findings: A retrospective study collected Hospital Consumer Assessment of Healthcare Providers and Systems (HCAHPS) surveys from 17,727 patients in 69 hospitals within the United States over a 14-month period.10 Of the 10 care dimensions analyzed, child comfort (aOR 1.50; 95% CI, 1.41-1.60) and nurse-parent communication (aOR 1.50; 95% CI, 1.42-1.58) were most strongly associated with a family’s willingness to recommend a hospital. Additional associated indices included preparing to leave the hospital (aOR 1.34; 95% CI, 1.27-1.41), doctor-parent communication (aOR 1.28; 95% CI, 1.21–1.35), and keeping parents informed (aOR 1.25; 95% CI, 1.18-1.33). Privacy and quietness, which are associated with positive patient experience in adult studies, were not significantly associated with willingness to recommend in this cohort.

Impact to practice: Hospitals seeking to improve patient experience will benefit most by focusing on improving patient comfort and nurse-parent communication. Factors that increase adult patient satisfaction may not be as important to the pediatric population and their families.



The other five articles that comprised the top 10 are listed below:
 

Comparison of as-needed and scheduled posthospitalization follow-up for children hospitalized for bronchiolitis

Coon ER et al. JAMA Pediatrics. September 2020.12

Clinical prediction rule for distinguishing bacterial from aseptic meningitis

Mintegi S et al. Pediatrics. September 2020.13

The Michigan Appropriateness Guide for Intravenous Catheters in Pediatrics: miniMAGIC Ullman AJ et al. Pediatrics. June 2020.14

A structured neonatal parenting elective: An approach for parenting leave during residency

Cree-Green M et al. Academic Pediatrics. Aug 2020.15

The KidzMed project: Teaching children to swallow tablet medication

Tse Y et al. Archives of Disease in Childhood. November 2020.16

Dr. Steed is an internal medicine and pediatrics hospitalist at Northwestern Memorial Hospital and Ann and Robert H. Lurie’s Children’s Hospital of Chicago. Dr. Fisher is a current fellow in hospice and palliative medicine and a clinical assistant professor at Michigan State University. Dr. Money is an assistant professor of pediatrics at the University of Utah and a fellowship-trained pediatric hospitalist at Utah Valley Hospital and Primary Children’s Hospital.

References

1. Khan A et al. Association between parent comfort with english and adverse events among hospitalized children. JAMA Pediatr. 2020 Dec 1;174(12):e203215. doi: 10.1001/jamapediatrics.2020.3215.

2. Wasserman M et al. Identifying and preventing medical errors in patients with limited English proficiency: Key findings and tools for the field. J Healthc Qual. May-Jun 2014;36(3):5-16. doi: 10.1111/jhq.12065.

3. Yeung F et al. Saline-lock versus continuous infusion: Maintaining peripheral intravenous catheter access in children. Hosp Pediatr. 2020 Dec;10(12):1038-43. doi: 10.1542/hpeds.2020-0137.

4. Mok E et al. A randomized controlled trial for maintaining peripheral intravenous lock in children. Int J Nurs Pract. 2007 Feb;13(1):33-45. doi: 10.1111/j.1440-172X.2006.00607.x.

5. Coon ER et al. Intensive care unit utilization after adoption of a ward-based high-flow nasal cannula protocol. J Hosp Med. 2020 Jun;15(6):325-30. doi: 10.12788/jhm.3417.

6. Kalburgi S and Halley T. High-flow nasal cannula use outside of the ICU setting. Pediatrics. 2020;146(5):e20194083. doi: 10.1542/peds.2019-4083.

7. Leyenaar JK and Ralston SL. Widespread adoption of low-value therapy: The case of bronchiolitis and high-flow oxygen. Pediatrics. 2020 Nov;146(5):e2020021188. doi: 10.1542/peds.2020-021188.

8. Van Kempen AAMW et al. Lower versus traditional treatment threshold for neonatal hypoglycemia. N Engl J Med. 2020 Feb 6;382(6):534-44. doi: 10.1056/NEJMoa1905593.

9. Shah R et al. Neonatal glycaemia and neurodevelopmental outcomes: A systematic review and meta-analysis. Neonatology. 2019;115(2):116-26. doi: 10.1159/000492859.

10. Feng JY et al. Factors associated with family experience in pediatric inpatient care. Pediatrics. 2020 Mar;145(3):e20191264. doi: 10.1542/peds.2019-1264.

11. Anhang Price R et al. Examining the role of patient experience surveys in measuring health care quality. Med Care Res Rev. 2014 Oct;71(5):522-54. doi: 10.1177/1077558714541480.

12. Coon ER et al. Comparison of as-needed and scheduled posthospitalization follow-up for children hospitalized for bronchiolitis: The Bronchiolitis Follow-up Intervention Trial (BeneFIT) randomized clinical trial. JAMA Pediatr. 2020 Sep 1;174(9):e201937. doi: 10.1001/jamapediatrics.2020.1937.

13. Mintegi S et al. Clinical prediction rule for distinguishing bacterial from aseptic meningitis. Pediatrics. 2020 Sept;146(3): e20201126. doi: 10.1542/peds.2020-1126.

14. Ullman AJ et al. The Michigan Appropriateness Guide for Intravenous Catheters in pediatrics: miniMAGIC. Pediatrics. 2020 Jun;145(Suppl 3):S269-S284. doi: 10.1542/peds.2019-3474I.

15. Cree-Green M et al. A structured neonatal parenting elective: an approach for parenting leave during residency. Acad Pediatr. 2021 Jan-Feb;21(1):16-18. doi: 10.1016/j.acap.2020.02.008.

16. Tse Y et al. The KidzMed project: Teaching children to swallow tablet medication. Arch Dis Child. 2020 Nov;105(11):1105-7. doi: 10.1136/archdischild-2019-317512.

Publications
Publications
Topics
Article Type
Sections
Disallow All Ads
Content Gating
No Gating (article Unlocked/Free)
Alternative CME
Disqus Comments
Default
Use ProPublica
Hide sidebar & use full width
render the right sidebar.
Conference Recap Checkbox
Not Conference Recap
Clinical Edge
Display the Slideshow in this Article
Medscape Article
Display survey writer
Reuters content
Disable Inline Native ads
WebMD Article

Timing of initiation of renal-replacement therapy in acute kidney injury

Article Type
Changed
Wed, 12/08/2021 - 14:28

Background: Acute kidney injury (AKI) is a common complication that occurs in seriously ill patients admitted to the ICU, and many of these patients eventually require RRT. When complicated by major metabolic disorders, it is usually clear when therapy should be initiated. However, when these complications are absent, the most appropriate time to initiate RRT is unclear. There are potential advantages to performing early RRT in patients with severe AKI, such as restoring acid-base balance, preventing fluid accumulation, and preventing major electrolyte disturbances.

Dr. Andrew Kim, division of hospital medicine, Mount Sinai Health System, New York
Dr. Andrew Kim


Study design: Multinational, randomized, controlled trial.

Setting: 168 hospitals in 15 countries.

Synopsis: Eligible patients were adults admitted to an ICU with severe AKI. Patients were randomly assigned to an accelerated strategy of RRT (initiated within 12 hours, 1,465 patients) or a standard strategy of RRT (held until conventional indications developed or AKI lasted more than 72 hours, 1,462 patients). RRT was performed in 1,418 (96.8%) in the accelerated group and 903 (61.8%) in the standard group. At 90 days, 643 deaths (43.9%) occurred in the accelerated group and 639 deaths (43.7%) occurred in the standard group (RR, 1.00; 95% CI, 0.93-1.09; P = .92). Among survivors at 90 days, 85 out of 814 accelerated patients (10.4%) and 49 of 815 standard patients (6.0%) continued to require RRT (RR, 1.75; 95% CI, 1.24-2.43), suggesting the possibility of increased dependence on long-term RRT if introduced early. Limitations include use of clinical equipoise to confirm full eligibility, introducing possible patient heterogeneity into the trial. In addition, broad discretion was given to clinicians on when to start RRT in the standard group resulting in variable initiation times.

Bottom line: In critically ill patients with severe AKI, earlier RRT did not result in lower mortality at 90 days compared with standard therapy and increased the risk of requiring RRT at 90 days.

Citation: Bagshaw SM et al. Timing of initiation of renal-replacement therapy in acute kidney injury. N Engl J Med. 2020;383:240-51. doi: 10.1056/NEJMoa2000741.

Dr. Kim is a hospitalist in the Division of Hospital Medicine, Mount Sinai Health System, New York.

Publications
Topics
Sections

Background: Acute kidney injury (AKI) is a common complication that occurs in seriously ill patients admitted to the ICU, and many of these patients eventually require RRT. When complicated by major metabolic disorders, it is usually clear when therapy should be initiated. However, when these complications are absent, the most appropriate time to initiate RRT is unclear. There are potential advantages to performing early RRT in patients with severe AKI, such as restoring acid-base balance, preventing fluid accumulation, and preventing major electrolyte disturbances.

Dr. Andrew Kim, division of hospital medicine, Mount Sinai Health System, New York
Dr. Andrew Kim


Study design: Multinational, randomized, controlled trial.

Setting: 168 hospitals in 15 countries.

Synopsis: Eligible patients were adults admitted to an ICU with severe AKI. Patients were randomly assigned to an accelerated strategy of RRT (initiated within 12 hours, 1,465 patients) or a standard strategy of RRT (held until conventional indications developed or AKI lasted more than 72 hours, 1,462 patients). RRT was performed in 1,418 (96.8%) in the accelerated group and 903 (61.8%) in the standard group. At 90 days, 643 deaths (43.9%) occurred in the accelerated group and 639 deaths (43.7%) occurred in the standard group (RR, 1.00; 95% CI, 0.93-1.09; P = .92). Among survivors at 90 days, 85 out of 814 accelerated patients (10.4%) and 49 of 815 standard patients (6.0%) continued to require RRT (RR, 1.75; 95% CI, 1.24-2.43), suggesting the possibility of increased dependence on long-term RRT if introduced early. Limitations include use of clinical equipoise to confirm full eligibility, introducing possible patient heterogeneity into the trial. In addition, broad discretion was given to clinicians on when to start RRT in the standard group resulting in variable initiation times.

Bottom line: In critically ill patients with severe AKI, earlier RRT did not result in lower mortality at 90 days compared with standard therapy and increased the risk of requiring RRT at 90 days.

Citation: Bagshaw SM et al. Timing of initiation of renal-replacement therapy in acute kidney injury. N Engl J Med. 2020;383:240-51. doi: 10.1056/NEJMoa2000741.

Dr. Kim is a hospitalist in the Division of Hospital Medicine, Mount Sinai Health System, New York.

Background: Acute kidney injury (AKI) is a common complication that occurs in seriously ill patients admitted to the ICU, and many of these patients eventually require RRT. When complicated by major metabolic disorders, it is usually clear when therapy should be initiated. However, when these complications are absent, the most appropriate time to initiate RRT is unclear. There are potential advantages to performing early RRT in patients with severe AKI, such as restoring acid-base balance, preventing fluid accumulation, and preventing major electrolyte disturbances.

Dr. Andrew Kim, division of hospital medicine, Mount Sinai Health System, New York
Dr. Andrew Kim


Study design: Multinational, randomized, controlled trial.

Setting: 168 hospitals in 15 countries.

Synopsis: Eligible patients were adults admitted to an ICU with severe AKI. Patients were randomly assigned to an accelerated strategy of RRT (initiated within 12 hours, 1,465 patients) or a standard strategy of RRT (held until conventional indications developed or AKI lasted more than 72 hours, 1,462 patients). RRT was performed in 1,418 (96.8%) in the accelerated group and 903 (61.8%) in the standard group. At 90 days, 643 deaths (43.9%) occurred in the accelerated group and 639 deaths (43.7%) occurred in the standard group (RR, 1.00; 95% CI, 0.93-1.09; P = .92). Among survivors at 90 days, 85 out of 814 accelerated patients (10.4%) and 49 of 815 standard patients (6.0%) continued to require RRT (RR, 1.75; 95% CI, 1.24-2.43), suggesting the possibility of increased dependence on long-term RRT if introduced early. Limitations include use of clinical equipoise to confirm full eligibility, introducing possible patient heterogeneity into the trial. In addition, broad discretion was given to clinicians on when to start RRT in the standard group resulting in variable initiation times.

Bottom line: In critically ill patients with severe AKI, earlier RRT did not result in lower mortality at 90 days compared with standard therapy and increased the risk of requiring RRT at 90 days.

Citation: Bagshaw SM et al. Timing of initiation of renal-replacement therapy in acute kidney injury. N Engl J Med. 2020;383:240-51. doi: 10.1056/NEJMoa2000741.

Dr. Kim is a hospitalist in the Division of Hospital Medicine, Mount Sinai Health System, New York.

Publications
Publications
Topics
Article Type
Sections
Disallow All Ads
Content Gating
No Gating (article Unlocked/Free)
Alternative CME
Disqus Comments
Default
Use ProPublica
Hide sidebar & use full width
render the right sidebar.
Conference Recap Checkbox
Not Conference Recap
Clinical Edge
Display the Slideshow in this Article
Medscape Article
Display survey writer
Reuters content
Disable Inline Native ads
WebMD Article

Children and COVID-19: 7 million cases and still counting

Article Type
Changed
Tue, 12/07/2021 - 15:16

Total COVID-19 cases in children surpassed the 7-million mark as new cases rose slightly after the previous week’s decline, according to the American Academy of Pediatrics and the Children’s Hospital Association.

Number of weekly COVID-19 cases reported in children, United States

The 133,000 new cases reported for the week ending Dec. 2 mark the 17th consecutive week that the count has exceeded 100,000 and brought the cumulative national count to 7.03 million since the start of the pandemic, the AAP and CHA said in their weekly COVID-19 report. New cases had dropped the previous week after 3 straight weeks of increases since late October.

The Centers for Disease Control and Prevention puts the total number of child COVID-19 cases at 6.2 million, but both estimates are based on all-age totals – 40 million for the CDC and 41 million for the AAP/CHA – that are well short of the CDC’s latest cumulative figure, which is now just over 49 million, so the actual figures are undoubtedly higher.

Meanwhile, the 1-month anniversary of 5- to 11-year-olds’ vaccine eligibility brought many completions: 923,000 received their second dose during the week ending Dec. 6, compared with 405,000 the previous week. About 16.9% (4.9 million) of children aged 5-11 have gotten at least one dose of the COVID-19 vaccine thus far, of whom almost 1.5 million children (5.1% of the age group) are now fully vaccinated, the CDC said on its COVID-19 Data Tracker.



The pace of vaccinations, however, is much lower for older children. Weekly numbers for all COVID-19 vaccinations, both first and second doses, dropped from 84,000 (Nov. 23-29) to 70,000 (Nov. 30 to Dec. 6), for those aged 12-17 years. In that group, 61.6% have received at least one dose and 51.8% are fully vaccinated, the CDC said.

The pace of vaccinations varies for younger children as well, when geography is considered. The AAP analyzed the CDC’s data and found that 42% of all 5- to 11-year-olds in Vermont had received at least one dose as of Dec. 1, followed by Massachusetts (33%), Maine (30%), and Rhode Island (28%). At the other end of the vaccination scale are Alabama, Louisiana, Mississippi, and West Virginia, all with 4%, the AAP reported.

As the United States puts 7 million children infected with COVID-19 in its rear view mirror, another milestone is looming ahead: The CDC’s current count of deaths in children is 974.

Publications
Topics
Sections

Total COVID-19 cases in children surpassed the 7-million mark as new cases rose slightly after the previous week’s decline, according to the American Academy of Pediatrics and the Children’s Hospital Association.

Number of weekly COVID-19 cases reported in children, United States

The 133,000 new cases reported for the week ending Dec. 2 mark the 17th consecutive week that the count has exceeded 100,000 and brought the cumulative national count to 7.03 million since the start of the pandemic, the AAP and CHA said in their weekly COVID-19 report. New cases had dropped the previous week after 3 straight weeks of increases since late October.

The Centers for Disease Control and Prevention puts the total number of child COVID-19 cases at 6.2 million, but both estimates are based on all-age totals – 40 million for the CDC and 41 million for the AAP/CHA – that are well short of the CDC’s latest cumulative figure, which is now just over 49 million, so the actual figures are undoubtedly higher.

Meanwhile, the 1-month anniversary of 5- to 11-year-olds’ vaccine eligibility brought many completions: 923,000 received their second dose during the week ending Dec. 6, compared with 405,000 the previous week. About 16.9% (4.9 million) of children aged 5-11 have gotten at least one dose of the COVID-19 vaccine thus far, of whom almost 1.5 million children (5.1% of the age group) are now fully vaccinated, the CDC said on its COVID-19 Data Tracker.



The pace of vaccinations, however, is much lower for older children. Weekly numbers for all COVID-19 vaccinations, both first and second doses, dropped from 84,000 (Nov. 23-29) to 70,000 (Nov. 30 to Dec. 6), for those aged 12-17 years. In that group, 61.6% have received at least one dose and 51.8% are fully vaccinated, the CDC said.

The pace of vaccinations varies for younger children as well, when geography is considered. The AAP analyzed the CDC’s data and found that 42% of all 5- to 11-year-olds in Vermont had received at least one dose as of Dec. 1, followed by Massachusetts (33%), Maine (30%), and Rhode Island (28%). At the other end of the vaccination scale are Alabama, Louisiana, Mississippi, and West Virginia, all with 4%, the AAP reported.

As the United States puts 7 million children infected with COVID-19 in its rear view mirror, another milestone is looming ahead: The CDC’s current count of deaths in children is 974.

Total COVID-19 cases in children surpassed the 7-million mark as new cases rose slightly after the previous week’s decline, according to the American Academy of Pediatrics and the Children’s Hospital Association.

Number of weekly COVID-19 cases reported in children, United States

The 133,000 new cases reported for the week ending Dec. 2 mark the 17th consecutive week that the count has exceeded 100,000 and brought the cumulative national count to 7.03 million since the start of the pandemic, the AAP and CHA said in their weekly COVID-19 report. New cases had dropped the previous week after 3 straight weeks of increases since late October.

The Centers for Disease Control and Prevention puts the total number of child COVID-19 cases at 6.2 million, but both estimates are based on all-age totals – 40 million for the CDC and 41 million for the AAP/CHA – that are well short of the CDC’s latest cumulative figure, which is now just over 49 million, so the actual figures are undoubtedly higher.

Meanwhile, the 1-month anniversary of 5- to 11-year-olds’ vaccine eligibility brought many completions: 923,000 received their second dose during the week ending Dec. 6, compared with 405,000 the previous week. About 16.9% (4.9 million) of children aged 5-11 have gotten at least one dose of the COVID-19 vaccine thus far, of whom almost 1.5 million children (5.1% of the age group) are now fully vaccinated, the CDC said on its COVID-19 Data Tracker.



The pace of vaccinations, however, is much lower for older children. Weekly numbers for all COVID-19 vaccinations, both first and second doses, dropped from 84,000 (Nov. 23-29) to 70,000 (Nov. 30 to Dec. 6), for those aged 12-17 years. In that group, 61.6% have received at least one dose and 51.8% are fully vaccinated, the CDC said.

The pace of vaccinations varies for younger children as well, when geography is considered. The AAP analyzed the CDC’s data and found that 42% of all 5- to 11-year-olds in Vermont had received at least one dose as of Dec. 1, followed by Massachusetts (33%), Maine (30%), and Rhode Island (28%). At the other end of the vaccination scale are Alabama, Louisiana, Mississippi, and West Virginia, all with 4%, the AAP reported.

As the United States puts 7 million children infected with COVID-19 in its rear view mirror, another milestone is looming ahead: The CDC’s current count of deaths in children is 974.

Publications
Publications
Topics
Article Type
Sections
Disallow All Ads
Content Gating
No Gating (article Unlocked/Free)
Alternative CME
Disqus Comments
Default
Use ProPublica
Hide sidebar & use full width
render the right sidebar.
Conference Recap Checkbox
Not Conference Recap
Clinical Edge
Display the Slideshow in this Article
Medscape Article
Display survey writer
Reuters content
Disable Inline Native ads
WebMD Article

Compression therapy prevents recurrence of cellulitis

Article Type
Changed
Tue, 12/07/2021 - 12:24

Background: Recurrent cellulitis is a common condition in patients with lower-extremity edema. Although some clinicians recommend compression garments as a preventative treatment, there are no data evaluating their efficacy for this purpose.

Dr. Michael Herscher. division of hospital medicine, Mount Sinai Health System, New York
Dr. Michael Herscher


Study design: Participants were randomized to receive either education alone or education plus compression therapy. Neither the participants nor the assessors were blinded to the treatment arm.

Setting: Single-center study in Australia.

Synopsis: Participants with cellulitis who also had at least two previous episodes of cellulitis in the previous 2 years and had lower-extremity edema were enrolled. Of participants, 84 were randomized. Both groups received education regarding skin care, body weight, and exercise, while the compression therapy group also received compression garments and instructions for their use. The primary outcome was recurrent cellulitis. Patients in the control group were allowed to cross over after an episode of cellulitis. The trial was stopped early for efficacy. At the time the trial was halted, 17 of 43 (40%) participants in the control group had recurrent cellulitis, compared with only 6 of 41 (15%) in the intervention (hazard ratio, 0.23; 95% CI, 0.09-0.59; P = .002). Limitations include the lack of blinding, which could have introduced bias, although the diagnosis of recurrent cellulitis was made by clinicians external to the trial. This study supports the use of compression garments in preventing recurrent cellulitis in patients with lower-extremity edema.

Bottom line: Compression garments can be used to prevent recurrent cellulitis in patients with edema.

Citation: Webb E et al. Compression therapy to prevent recurrent cellulitis of the leg. N Engl J Med. 2020;383(7):630-9. doi:10.1056/NEJMoa1917197.

Dr. Herscher is a hospitalist in the Division of Hospital Medicine, Mount Sinai Health System, New York.

Publications
Topics
Sections

Background: Recurrent cellulitis is a common condition in patients with lower-extremity edema. Although some clinicians recommend compression garments as a preventative treatment, there are no data evaluating their efficacy for this purpose.

Dr. Michael Herscher. division of hospital medicine, Mount Sinai Health System, New York
Dr. Michael Herscher


Study design: Participants were randomized to receive either education alone or education plus compression therapy. Neither the participants nor the assessors were blinded to the treatment arm.

Setting: Single-center study in Australia.

Synopsis: Participants with cellulitis who also had at least two previous episodes of cellulitis in the previous 2 years and had lower-extremity edema were enrolled. Of participants, 84 were randomized. Both groups received education regarding skin care, body weight, and exercise, while the compression therapy group also received compression garments and instructions for their use. The primary outcome was recurrent cellulitis. Patients in the control group were allowed to cross over after an episode of cellulitis. The trial was stopped early for efficacy. At the time the trial was halted, 17 of 43 (40%) participants in the control group had recurrent cellulitis, compared with only 6 of 41 (15%) in the intervention (hazard ratio, 0.23; 95% CI, 0.09-0.59; P = .002). Limitations include the lack of blinding, which could have introduced bias, although the diagnosis of recurrent cellulitis was made by clinicians external to the trial. This study supports the use of compression garments in preventing recurrent cellulitis in patients with lower-extremity edema.

Bottom line: Compression garments can be used to prevent recurrent cellulitis in patients with edema.

Citation: Webb E et al. Compression therapy to prevent recurrent cellulitis of the leg. N Engl J Med. 2020;383(7):630-9. doi:10.1056/NEJMoa1917197.

Dr. Herscher is a hospitalist in the Division of Hospital Medicine, Mount Sinai Health System, New York.

Background: Recurrent cellulitis is a common condition in patients with lower-extremity edema. Although some clinicians recommend compression garments as a preventative treatment, there are no data evaluating their efficacy for this purpose.

Dr. Michael Herscher. division of hospital medicine, Mount Sinai Health System, New York
Dr. Michael Herscher


Study design: Participants were randomized to receive either education alone or education plus compression therapy. Neither the participants nor the assessors were blinded to the treatment arm.

Setting: Single-center study in Australia.

Synopsis: Participants with cellulitis who also had at least two previous episodes of cellulitis in the previous 2 years and had lower-extremity edema were enrolled. Of participants, 84 were randomized. Both groups received education regarding skin care, body weight, and exercise, while the compression therapy group also received compression garments and instructions for their use. The primary outcome was recurrent cellulitis. Patients in the control group were allowed to cross over after an episode of cellulitis. The trial was stopped early for efficacy. At the time the trial was halted, 17 of 43 (40%) participants in the control group had recurrent cellulitis, compared with only 6 of 41 (15%) in the intervention (hazard ratio, 0.23; 95% CI, 0.09-0.59; P = .002). Limitations include the lack of blinding, which could have introduced bias, although the diagnosis of recurrent cellulitis was made by clinicians external to the trial. This study supports the use of compression garments in preventing recurrent cellulitis in patients with lower-extremity edema.

Bottom line: Compression garments can be used to prevent recurrent cellulitis in patients with edema.

Citation: Webb E et al. Compression therapy to prevent recurrent cellulitis of the leg. N Engl J Med. 2020;383(7):630-9. doi:10.1056/NEJMoa1917197.

Dr. Herscher is a hospitalist in the Division of Hospital Medicine, Mount Sinai Health System, New York.

Publications
Publications
Topics
Article Type
Sections
Disallow All Ads
Content Gating
No Gating (article Unlocked/Free)
Alternative CME
Disqus Comments
Default
Use ProPublica
Hide sidebar & use full width
render the right sidebar.
Conference Recap Checkbox
Not Conference Recap
Clinical Edge
Display the Slideshow in this Article
Medscape Article
Display survey writer
Reuters content
Disable Inline Native ads
WebMD Article

Apixaban outmatches rivaroxaban for VTE in study

Article Type
Changed
Thu, 12/09/2021 - 14:48

 

Apixaban appears to be safer and more effective than rivaroxaban for reducing risk of venous thromboembolism and bleeding, based on new research.

Recurrent venous thromboembolism (VTE) – a composite of pulmonary embolism and deep vein thrombosis – was the primary effectiveness outcome in the retrospective analysis of new-user data from almost 40,000 patients, which was published in Annals of Internal Medicine. Safety was evaluated through a composite of intracranial and gastrointestinal bleeding.

After a median follow-up of 102 days in the apixaban group and 105 days in the rivaroxaban group, apixaban demonstrated superiority for both primary outcomes.

These real-world findings may guide selection of initial anticoagulant therapy, reported lead author Ghadeer K. Dawwas, PhD, MSc, MBA, of the University of Pennsylvania, Philadelphia, and colleagues.

“Randomized clinical trials comparing apixaban with rivaroxaban in patients with VTE are under way (for example, COBRRA (NCT03266783),” the investigators wrote. “Until the results from these trials become available (The estimated completion date for COBRRA is December 2023.), observational studies that use existing data can provide evidence on the effectiveness and safety of these alternatives to inform clinical practice.”

In the new research, apixaban was associated with a 23% lower rate of recurrent VTE (hazard ratio, 0.77; 95% confidence interval, 0.69-0.87), including a 15% lower rate of deep vein thrombosis and a 41% lower rate of pulmonary embolism. Apixaban was associated with 40% fewer bleeding events (HR, 0.60; 95% CI, 0.53-0.69]), including a 40% lower rate of GI bleeding and a 46% lower rate of intracranial bleeding.

The study involved 37,236 patients with VTE, all of whom were diagnosed in at least one inpatient encounter and initiated direct oral anticoagulant (DOAC) therapy within 30 days, according to Optum’s deidentified Clinformatics Data Mart Database. Patients were evenly split into apixaban and rivaroxaban groups, with 18,618 individuals in each. Propensity score matching was used to minimize differences in baseline characteristics.

Apixaban was associated with an absolute reduction in recurrent VTE of 0.6% and 1.1% over 2 and 6 months, respectively, as well as reductions in bleeding of 1.1% and 1.5% over the same respective time periods.

The investigators noted that these findings were maintained in various sensitivity and subgroup analyses, including a model in which patients with VTE who had transient risk factors were compared with VTE patients exhibiting chronic risk factors.

“These findings suggest that apixaban has superior effectiveness and safety, compared with rivaroxaban and may provide guidance to clinicians and patients regarding selection of an anticoagulant for treatment of VTE,” Dr. Dawwas and colleagues concluded.

Study may have missed some nuance in possible outcomes, according to vascular surgeon

Thomas Wakefield, MD, a vascular surgeon and a professor of surgery at the University of Michigan Health Frankel Cardiovascular Center, Ann Arbor, generally agreed with the investigators’ conclusion, although he noted that DOAC selection may also be influenced by other considerations.

Dr. Damon E. Houghton

“The results of this study suggest that, when choosing an agent for an individual patient, apixaban does appear to have an advantage over rivaroxaban related to recurrent VTE and bleeding,” Dr. Wakefield said in an interview. “One must keep in mind that these are not the only factors that are considered when choosing an agent and these are not the only two DOACs available. For example, rivaroxaban is given once per day while apixaban is given twice per day, and rivaroxaban has been shown to be successful in the treatment of other thrombotic disorders.”

Dr. Wakefield also pointed out that the study may have missed some nuance in possible outcomes.

“The current study looked at severe outcomes that resulted in inpatient hospitalization, so the generalization to strictly outpatient treatment and less severe outcomes cannot be inferred,” he said.

Damon E. Houghton, MD, of the department of medicine and a consultant in the department of vascular medicine and hematology at Mayo Clinic, Rochester, Minn., called the study a “very nice analysis,” highlighting the large sample size.

“The results are not a reason to abandon rivaroxaban altogether, but do suggest that, when otherwise appropriate for a patient, apixaban should be the first choice,” Dr. Houghton said in a written comment. “Hopefully this analysis will encourage more payers to create financial incentives that facilitate the use of apixaban in more patients.”

 

 

Randomized trial needed, says hematologist

Colleen Edwards, MD, of the departments of medicine, hematology, and medical oncology, at the Icahn School of Medicine at Mount Sinai, New York, had a more guarded view of the findings than Dr. Wakefield and Dr. Houghton.

Dr. Colleen Edwards

“[The investigators] certainly seem to be doing a lot of statistical gymnastics in this paper,” Dr. Edwards said in an interview. “They used all kinds of surrogates in place of real data that you would get from a randomized trial.”

For example, Dr. Edwards noted the use of prescription refills as a surrogate for medication adherence, and emphasized that inpatient observational data may not reflect outpatient therapy.

“Inpatients are constantly missing their medicines all the time,” she said. “They’re holding it for procedures, they’re NPO, they’re off the floor, so they missed their medicine. So it’s just a very different patient population than the outpatient population, which is where venous thromboembolism is treated now, by and large.”

Although Dr. Edwards suggested that the findings might guide treatment selection “a little bit,” she noted that insurance constraints and costs play a greater role, and ultimately concluded that a randomized trial is needed to materially alter clinical decision-making.

“I think we really have to wait for randomized trial before we abandon our other choices,” she said.

The investigators disclosed relationships with Merck, Celgene, UCB, and others. Dr. Wakefield reported awaiting disclosures. Dr. Houghton and Dr. Edwards reported no relevant conflicts of interest.

Publications
Topics
Sections

 

Apixaban appears to be safer and more effective than rivaroxaban for reducing risk of venous thromboembolism and bleeding, based on new research.

Recurrent venous thromboembolism (VTE) – a composite of pulmonary embolism and deep vein thrombosis – was the primary effectiveness outcome in the retrospective analysis of new-user data from almost 40,000 patients, which was published in Annals of Internal Medicine. Safety was evaluated through a composite of intracranial and gastrointestinal bleeding.

After a median follow-up of 102 days in the apixaban group and 105 days in the rivaroxaban group, apixaban demonstrated superiority for both primary outcomes.

These real-world findings may guide selection of initial anticoagulant therapy, reported lead author Ghadeer K. Dawwas, PhD, MSc, MBA, of the University of Pennsylvania, Philadelphia, and colleagues.

“Randomized clinical trials comparing apixaban with rivaroxaban in patients with VTE are under way (for example, COBRRA (NCT03266783),” the investigators wrote. “Until the results from these trials become available (The estimated completion date for COBRRA is December 2023.), observational studies that use existing data can provide evidence on the effectiveness and safety of these alternatives to inform clinical practice.”

In the new research, apixaban was associated with a 23% lower rate of recurrent VTE (hazard ratio, 0.77; 95% confidence interval, 0.69-0.87), including a 15% lower rate of deep vein thrombosis and a 41% lower rate of pulmonary embolism. Apixaban was associated with 40% fewer bleeding events (HR, 0.60; 95% CI, 0.53-0.69]), including a 40% lower rate of GI bleeding and a 46% lower rate of intracranial bleeding.

The study involved 37,236 patients with VTE, all of whom were diagnosed in at least one inpatient encounter and initiated direct oral anticoagulant (DOAC) therapy within 30 days, according to Optum’s deidentified Clinformatics Data Mart Database. Patients were evenly split into apixaban and rivaroxaban groups, with 18,618 individuals in each. Propensity score matching was used to minimize differences in baseline characteristics.

Apixaban was associated with an absolute reduction in recurrent VTE of 0.6% and 1.1% over 2 and 6 months, respectively, as well as reductions in bleeding of 1.1% and 1.5% over the same respective time periods.

The investigators noted that these findings were maintained in various sensitivity and subgroup analyses, including a model in which patients with VTE who had transient risk factors were compared with VTE patients exhibiting chronic risk factors.

“These findings suggest that apixaban has superior effectiveness and safety, compared with rivaroxaban and may provide guidance to clinicians and patients regarding selection of an anticoagulant for treatment of VTE,” Dr. Dawwas and colleagues concluded.

Study may have missed some nuance in possible outcomes, according to vascular surgeon

Thomas Wakefield, MD, a vascular surgeon and a professor of surgery at the University of Michigan Health Frankel Cardiovascular Center, Ann Arbor, generally agreed with the investigators’ conclusion, although he noted that DOAC selection may also be influenced by other considerations.

Dr. Damon E. Houghton

“The results of this study suggest that, when choosing an agent for an individual patient, apixaban does appear to have an advantage over rivaroxaban related to recurrent VTE and bleeding,” Dr. Wakefield said in an interview. “One must keep in mind that these are not the only factors that are considered when choosing an agent and these are not the only two DOACs available. For example, rivaroxaban is given once per day while apixaban is given twice per day, and rivaroxaban has been shown to be successful in the treatment of other thrombotic disorders.”

Dr. Wakefield also pointed out that the study may have missed some nuance in possible outcomes.

“The current study looked at severe outcomes that resulted in inpatient hospitalization, so the generalization to strictly outpatient treatment and less severe outcomes cannot be inferred,” he said.

Damon E. Houghton, MD, of the department of medicine and a consultant in the department of vascular medicine and hematology at Mayo Clinic, Rochester, Minn., called the study a “very nice analysis,” highlighting the large sample size.

“The results are not a reason to abandon rivaroxaban altogether, but do suggest that, when otherwise appropriate for a patient, apixaban should be the first choice,” Dr. Houghton said in a written comment. “Hopefully this analysis will encourage more payers to create financial incentives that facilitate the use of apixaban in more patients.”

 

 

Randomized trial needed, says hematologist

Colleen Edwards, MD, of the departments of medicine, hematology, and medical oncology, at the Icahn School of Medicine at Mount Sinai, New York, had a more guarded view of the findings than Dr. Wakefield and Dr. Houghton.

Dr. Colleen Edwards

“[The investigators] certainly seem to be doing a lot of statistical gymnastics in this paper,” Dr. Edwards said in an interview. “They used all kinds of surrogates in place of real data that you would get from a randomized trial.”

For example, Dr. Edwards noted the use of prescription refills as a surrogate for medication adherence, and emphasized that inpatient observational data may not reflect outpatient therapy.

“Inpatients are constantly missing their medicines all the time,” she said. “They’re holding it for procedures, they’re NPO, they’re off the floor, so they missed their medicine. So it’s just a very different patient population than the outpatient population, which is where venous thromboembolism is treated now, by and large.”

Although Dr. Edwards suggested that the findings might guide treatment selection “a little bit,” she noted that insurance constraints and costs play a greater role, and ultimately concluded that a randomized trial is needed to materially alter clinical decision-making.

“I think we really have to wait for randomized trial before we abandon our other choices,” she said.

The investigators disclosed relationships with Merck, Celgene, UCB, and others. Dr. Wakefield reported awaiting disclosures. Dr. Houghton and Dr. Edwards reported no relevant conflicts of interest.

 

Apixaban appears to be safer and more effective than rivaroxaban for reducing risk of venous thromboembolism and bleeding, based on new research.

Recurrent venous thromboembolism (VTE) – a composite of pulmonary embolism and deep vein thrombosis – was the primary effectiveness outcome in the retrospective analysis of new-user data from almost 40,000 patients, which was published in Annals of Internal Medicine. Safety was evaluated through a composite of intracranial and gastrointestinal bleeding.

After a median follow-up of 102 days in the apixaban group and 105 days in the rivaroxaban group, apixaban demonstrated superiority for both primary outcomes.

These real-world findings may guide selection of initial anticoagulant therapy, reported lead author Ghadeer K. Dawwas, PhD, MSc, MBA, of the University of Pennsylvania, Philadelphia, and colleagues.

“Randomized clinical trials comparing apixaban with rivaroxaban in patients with VTE are under way (for example, COBRRA (NCT03266783),” the investigators wrote. “Until the results from these trials become available (The estimated completion date for COBRRA is December 2023.), observational studies that use existing data can provide evidence on the effectiveness and safety of these alternatives to inform clinical practice.”

In the new research, apixaban was associated with a 23% lower rate of recurrent VTE (hazard ratio, 0.77; 95% confidence interval, 0.69-0.87), including a 15% lower rate of deep vein thrombosis and a 41% lower rate of pulmonary embolism. Apixaban was associated with 40% fewer bleeding events (HR, 0.60; 95% CI, 0.53-0.69]), including a 40% lower rate of GI bleeding and a 46% lower rate of intracranial bleeding.

The study involved 37,236 patients with VTE, all of whom were diagnosed in at least one inpatient encounter and initiated direct oral anticoagulant (DOAC) therapy within 30 days, according to Optum’s deidentified Clinformatics Data Mart Database. Patients were evenly split into apixaban and rivaroxaban groups, with 18,618 individuals in each. Propensity score matching was used to minimize differences in baseline characteristics.

Apixaban was associated with an absolute reduction in recurrent VTE of 0.6% and 1.1% over 2 and 6 months, respectively, as well as reductions in bleeding of 1.1% and 1.5% over the same respective time periods.

The investigators noted that these findings were maintained in various sensitivity and subgroup analyses, including a model in which patients with VTE who had transient risk factors were compared with VTE patients exhibiting chronic risk factors.

“These findings suggest that apixaban has superior effectiveness and safety, compared with rivaroxaban and may provide guidance to clinicians and patients regarding selection of an anticoagulant for treatment of VTE,” Dr. Dawwas and colleagues concluded.

Study may have missed some nuance in possible outcomes, according to vascular surgeon

Thomas Wakefield, MD, a vascular surgeon and a professor of surgery at the University of Michigan Health Frankel Cardiovascular Center, Ann Arbor, generally agreed with the investigators’ conclusion, although he noted that DOAC selection may also be influenced by other considerations.

Dr. Damon E. Houghton

“The results of this study suggest that, when choosing an agent for an individual patient, apixaban does appear to have an advantage over rivaroxaban related to recurrent VTE and bleeding,” Dr. Wakefield said in an interview. “One must keep in mind that these are not the only factors that are considered when choosing an agent and these are not the only two DOACs available. For example, rivaroxaban is given once per day while apixaban is given twice per day, and rivaroxaban has been shown to be successful in the treatment of other thrombotic disorders.”

Dr. Wakefield also pointed out that the study may have missed some nuance in possible outcomes.

“The current study looked at severe outcomes that resulted in inpatient hospitalization, so the generalization to strictly outpatient treatment and less severe outcomes cannot be inferred,” he said.

Damon E. Houghton, MD, of the department of medicine and a consultant in the department of vascular medicine and hematology at Mayo Clinic, Rochester, Minn., called the study a “very nice analysis,” highlighting the large sample size.

“The results are not a reason to abandon rivaroxaban altogether, but do suggest that, when otherwise appropriate for a patient, apixaban should be the first choice,” Dr. Houghton said in a written comment. “Hopefully this analysis will encourage more payers to create financial incentives that facilitate the use of apixaban in more patients.”

 

 

Randomized trial needed, says hematologist

Colleen Edwards, MD, of the departments of medicine, hematology, and medical oncology, at the Icahn School of Medicine at Mount Sinai, New York, had a more guarded view of the findings than Dr. Wakefield and Dr. Houghton.

Dr. Colleen Edwards

“[The investigators] certainly seem to be doing a lot of statistical gymnastics in this paper,” Dr. Edwards said in an interview. “They used all kinds of surrogates in place of real data that you would get from a randomized trial.”

For example, Dr. Edwards noted the use of prescription refills as a surrogate for medication adherence, and emphasized that inpatient observational data may not reflect outpatient therapy.

“Inpatients are constantly missing their medicines all the time,” she said. “They’re holding it for procedures, they’re NPO, they’re off the floor, so they missed their medicine. So it’s just a very different patient population than the outpatient population, which is where venous thromboembolism is treated now, by and large.”

Although Dr. Edwards suggested that the findings might guide treatment selection “a little bit,” she noted that insurance constraints and costs play a greater role, and ultimately concluded that a randomized trial is needed to materially alter clinical decision-making.

“I think we really have to wait for randomized trial before we abandon our other choices,” she said.

The investigators disclosed relationships with Merck, Celgene, UCB, and others. Dr. Wakefield reported awaiting disclosures. Dr. Houghton and Dr. Edwards reported no relevant conflicts of interest.

Publications
Publications
Topics
Article Type
Sections
Article Source

FROM ANNALS OF INTERNAL MEDICINE

Disallow All Ads
Content Gating
No Gating (article Unlocked/Free)
Alternative CME
Disqus Comments
Default
Use ProPublica
Hide sidebar & use full width
render the right sidebar.
Conference Recap Checkbox
Not Conference Recap
Clinical Edge
Display the Slideshow in this Article
Medscape Article
Display survey writer
Reuters content
Disable Inline Native ads
WebMD Article

Care via video teleconferencing can be as effective as in-person for some conditions

Article Type
Changed
Tue, 05/03/2022 - 15:02

As the pandemic shows no signs of ending, primary care doctors may be reassured that delivering care via video teleconferencing can be as effective as usual in-person consultation for several common health conditions.

Jordan Albritton, PhD, RTI International in Research Triangle Park, N.C.
Dr. Jordan Albritton

This was a finding of a new study published in Annals of Internal Medicine involving a review of literature on video teleconferencing (VTC) visits, which was authored by Jordan Albritton, PhD, MPH and his colleagues.

The authors found generally comparable patient outcomes as well as no differences in health care use, patient satisfaction, and quality of life when visits conducted using VTC were compared with usual care.

While VTC may work best for monitoring patients with chronic conditions, it can also be effective for acute care, said Dr. Albritton, who is a research public health analyst at RTI International in Research Triangle Park, N.C., in an interview.

The investigators analyzed 20 randomized controlled trials of at least 50 patients and acceptable risk of bias in which VTC was used either for main or adjunct care delivery. Published from 2013 to 2019, these studies looked at care for diabetes and pain management, as well as some respiratory, neurologic, and cardiovascular conditions. Studies comparing VTC with usual care that did not involve any added in-person care were more likely to favor the VTC group, the investigators found.

“We excluded conditions such as substance use disorders, maternal care, and weight management for which there was sufficient prior evidence of the benefit of VTC,” Dr. Albritton said in an interview. “But I don’t think our results would have been substantially different if we had included these other diseases. We found general evidence in the literature that VTC is effective for a broader range of conditions.”

In some cases, such as if changes in a patient’s condition triggered an automatic virtual visit, the author said he thinks VTC may lead to even greater effectiveness.

“The doctor and patient could figure out on the spot what’s going on and perhaps change the medication,” Dr. Albritton explained.

In general agreement is Julia L. Frydman, MD, assistant professor in the Brookdale Department of Geriatric and Palliative Medicine at Icahn School of Medicine at Mount Sinai in New York, who was not involved in the RTI research.

Julia L. Frydman, MD, Icahn School of Medicine at Mount Sinai in New York
Dr. Julia L. Frydman

“Telemedicine has promise across many medical subspecialties, and what we need now are more studies to understand the perspectives of patients, caregivers, and clinicians as well as the impact of telemedicine on health outcomes and healthcare utilization.”

In acknowledgment of their utility, video visits are on the rise in the United States. A 2020 survey found that 22% of patients and 80% of physicians reported having participated in a video visit, three times the rate of the previous year. The authors noted that policy changes enacted to support telehealth strategies during the pandemic are expected to remain in place, and although patients are returning to in-person care, the virtual visit market will likely continue growing.
 

 

 

Increased telemedicine use by older adults

“We’ve seen an exciting expansion of telemedicine use among older adults, and we need to focus on continuing to meet their needs,” Dr. Frydman said.

In a recent study of televisits during the pandemic, Dr. Frydman’s group found a fivefold greater uptake of remote consultations by seniors – from 5% to 25%. Although in-person visits were far more common among older adults.

A specific advantage of video-based over audio-only telehealth, noted Dr. Albritton, is that physicians can directly observe patients in their home environment. Sharing that view is Deepa Iyengar, MBBS/MD,MPH, professor of family medicine at McGovern Medical School at The University of Texas Health Science Center at Houston, where, she said, “the pandemic has put VTC use into overdrive.”

Deepa Iyengar, MBBS/MD ,MPH, University of Texas Health Science Center at Houston
Dr. Deepa Iyengar

According to Dr. Iyengar, who was not involved in the RTI research, the video component definitely represents value-added over phone calls. “You can pick up visual cues on video that you might not see if the patient came in and you can see what the home environment is like – whether there are a lot of loose rugs on the floor or broken or missing light bulbs,” she said in an interview.
 

‘VTC is here to stay’

In other parts of the country, doctors are finding virtual care useful – and more common. “VTC is here to stay, for sure – the horse is out of the barn,” said Cheryl L. Wilkes, MD, an internist at Northwestern Medicine and assistant professor of medicine at Northwestern University in Chicago. “The RTI study shows no harm from VTC and also shows it may even improve clinical outcomes.”

Cheryl L. Wilkes, MD, Northwestern University, Chicago
Dr. Cheryl L. Wilkes

Video visits can also save patients high parking fees at clinics and spare the sick or elderly from having to hire caregivers to bring them into the office or from having to walk blocks in dangerous weather conditions, she added. “And I can do a virtual visit on the fly or at night when a relative or caregiver is home from work to be there with the patient.”

In addition to being beneficial for following up with patients with chronic diseases such as hypertension or diabetes, VTC may be able to replace some visits that have traditionally required hands-on care, said Dr. Wilkes.

She said she knows a cardiologist who has refined a process whereby a patient – say, one who may have edema – is asked to perform a maneuver via VTC and then display the result to the doctor: The doctor says, “put your leg up and press on it hard for 10 seconds and then show me what it looks like,” according to Dr. Wilkes.

The key now is to identify the best persons across specialties from neurology to rheumatology to videotape ways they’ve created to help their patients participate virtually in consults traditionally done at the office, Dr. Wilkes noted.

But some conditions will always require palpation and the use of a stethoscope, according Dr. Iyengar.

“If someone has an ulcer, I have to be able to feel it,” she said.

And while some maternity care can be given virtually – for instance, if a mother-to be develops a bad cold – hands-on obstetrical care to check the position and health of the baby obviously has to be done in person. “So VTC is definitely going to be a welcome addition but not a replacement,” Dr. Iyengar said.

Gaps in research on VTC visits

Many questions remain regarding the overall usefulness of VTC visits for certain patient groups, according to the authors.

They highlighted, for example, the dearth of data on subgroups or on underserved and vulnerable populations, with no head-to-head studies identified in their review. In addition, they found no studies examining VTC versus usual care for patients with concurrent conditions or on its effect on health equity and disparities.

“It’s now our job to understand the ongoing barriers to telemedicine access, including the digital divide and the usability of telemedicine platforms, and design interventions that overcome them,” Dr. Frydman said. “At the same time, we need to make sure we’re understanding and respecting the preferences of older adults in terms of how they access health care.”

This study was supported by the Patient-Centered Outcomes Research Institute (PCORI). Dr. Albritton is employed by RTI International, the contractor responsible for conducting the research and developing the manuscript. Several coauthors disclosed support from or contracts with PCORI. One coauthor’s spouse holds stock in private health companies. Dr. Frydman, Dr. Iyengar, and Dr. Wilkes disclosed no competing interests relevant to their comments.

Publications
Topics
Sections

As the pandemic shows no signs of ending, primary care doctors may be reassured that delivering care via video teleconferencing can be as effective as usual in-person consultation for several common health conditions.

Jordan Albritton, PhD, RTI International in Research Triangle Park, N.C.
Dr. Jordan Albritton

This was a finding of a new study published in Annals of Internal Medicine involving a review of literature on video teleconferencing (VTC) visits, which was authored by Jordan Albritton, PhD, MPH and his colleagues.

The authors found generally comparable patient outcomes as well as no differences in health care use, patient satisfaction, and quality of life when visits conducted using VTC were compared with usual care.

While VTC may work best for monitoring patients with chronic conditions, it can also be effective for acute care, said Dr. Albritton, who is a research public health analyst at RTI International in Research Triangle Park, N.C., in an interview.

The investigators analyzed 20 randomized controlled trials of at least 50 patients and acceptable risk of bias in which VTC was used either for main or adjunct care delivery. Published from 2013 to 2019, these studies looked at care for diabetes and pain management, as well as some respiratory, neurologic, and cardiovascular conditions. Studies comparing VTC with usual care that did not involve any added in-person care were more likely to favor the VTC group, the investigators found.

“We excluded conditions such as substance use disorders, maternal care, and weight management for which there was sufficient prior evidence of the benefit of VTC,” Dr. Albritton said in an interview. “But I don’t think our results would have been substantially different if we had included these other diseases. We found general evidence in the literature that VTC is effective for a broader range of conditions.”

In some cases, such as if changes in a patient’s condition triggered an automatic virtual visit, the author said he thinks VTC may lead to even greater effectiveness.

“The doctor and patient could figure out on the spot what’s going on and perhaps change the medication,” Dr. Albritton explained.

In general agreement is Julia L. Frydman, MD, assistant professor in the Brookdale Department of Geriatric and Palliative Medicine at Icahn School of Medicine at Mount Sinai in New York, who was not involved in the RTI research.

Julia L. Frydman, MD, Icahn School of Medicine at Mount Sinai in New York
Dr. Julia L. Frydman

“Telemedicine has promise across many medical subspecialties, and what we need now are more studies to understand the perspectives of patients, caregivers, and clinicians as well as the impact of telemedicine on health outcomes and healthcare utilization.”

In acknowledgment of their utility, video visits are on the rise in the United States. A 2020 survey found that 22% of patients and 80% of physicians reported having participated in a video visit, three times the rate of the previous year. The authors noted that policy changes enacted to support telehealth strategies during the pandemic are expected to remain in place, and although patients are returning to in-person care, the virtual visit market will likely continue growing.
 

 

 

Increased telemedicine use by older adults

“We’ve seen an exciting expansion of telemedicine use among older adults, and we need to focus on continuing to meet their needs,” Dr. Frydman said.

In a recent study of televisits during the pandemic, Dr. Frydman’s group found a fivefold greater uptake of remote consultations by seniors – from 5% to 25%. Although in-person visits were far more common among older adults.

A specific advantage of video-based over audio-only telehealth, noted Dr. Albritton, is that physicians can directly observe patients in their home environment. Sharing that view is Deepa Iyengar, MBBS/MD,MPH, professor of family medicine at McGovern Medical School at The University of Texas Health Science Center at Houston, where, she said, “the pandemic has put VTC use into overdrive.”

Deepa Iyengar, MBBS/MD ,MPH, University of Texas Health Science Center at Houston
Dr. Deepa Iyengar

According to Dr. Iyengar, who was not involved in the RTI research, the video component definitely represents value-added over phone calls. “You can pick up visual cues on video that you might not see if the patient came in and you can see what the home environment is like – whether there are a lot of loose rugs on the floor or broken or missing light bulbs,” she said in an interview.
 

‘VTC is here to stay’

In other parts of the country, doctors are finding virtual care useful – and more common. “VTC is here to stay, for sure – the horse is out of the barn,” said Cheryl L. Wilkes, MD, an internist at Northwestern Medicine and assistant professor of medicine at Northwestern University in Chicago. “The RTI study shows no harm from VTC and also shows it may even improve clinical outcomes.”

Cheryl L. Wilkes, MD, Northwestern University, Chicago
Dr. Cheryl L. Wilkes

Video visits can also save patients high parking fees at clinics and spare the sick or elderly from having to hire caregivers to bring them into the office or from having to walk blocks in dangerous weather conditions, she added. “And I can do a virtual visit on the fly or at night when a relative or caregiver is home from work to be there with the patient.”

In addition to being beneficial for following up with patients with chronic diseases such as hypertension or diabetes, VTC may be able to replace some visits that have traditionally required hands-on care, said Dr. Wilkes.

She said she knows a cardiologist who has refined a process whereby a patient – say, one who may have edema – is asked to perform a maneuver via VTC and then display the result to the doctor: The doctor says, “put your leg up and press on it hard for 10 seconds and then show me what it looks like,” according to Dr. Wilkes.

The key now is to identify the best persons across specialties from neurology to rheumatology to videotape ways they’ve created to help their patients participate virtually in consults traditionally done at the office, Dr. Wilkes noted.

But some conditions will always require palpation and the use of a stethoscope, according Dr. Iyengar.

“If someone has an ulcer, I have to be able to feel it,” she said.

And while some maternity care can be given virtually – for instance, if a mother-to be develops a bad cold – hands-on obstetrical care to check the position and health of the baby obviously has to be done in person. “So VTC is definitely going to be a welcome addition but not a replacement,” Dr. Iyengar said.

Gaps in research on VTC visits

Many questions remain regarding the overall usefulness of VTC visits for certain patient groups, according to the authors.

They highlighted, for example, the dearth of data on subgroups or on underserved and vulnerable populations, with no head-to-head studies identified in their review. In addition, they found no studies examining VTC versus usual care for patients with concurrent conditions or on its effect on health equity and disparities.

“It’s now our job to understand the ongoing barriers to telemedicine access, including the digital divide and the usability of telemedicine platforms, and design interventions that overcome them,” Dr. Frydman said. “At the same time, we need to make sure we’re understanding and respecting the preferences of older adults in terms of how they access health care.”

This study was supported by the Patient-Centered Outcomes Research Institute (PCORI). Dr. Albritton is employed by RTI International, the contractor responsible for conducting the research and developing the manuscript. Several coauthors disclosed support from or contracts with PCORI. One coauthor’s spouse holds stock in private health companies. Dr. Frydman, Dr. Iyengar, and Dr. Wilkes disclosed no competing interests relevant to their comments.

As the pandemic shows no signs of ending, primary care doctors may be reassured that delivering care via video teleconferencing can be as effective as usual in-person consultation for several common health conditions.

Jordan Albritton, PhD, RTI International in Research Triangle Park, N.C.
Dr. Jordan Albritton

This was a finding of a new study published in Annals of Internal Medicine involving a review of literature on video teleconferencing (VTC) visits, which was authored by Jordan Albritton, PhD, MPH and his colleagues.

The authors found generally comparable patient outcomes as well as no differences in health care use, patient satisfaction, and quality of life when visits conducted using VTC were compared with usual care.

While VTC may work best for monitoring patients with chronic conditions, it can also be effective for acute care, said Dr. Albritton, who is a research public health analyst at RTI International in Research Triangle Park, N.C., in an interview.

The investigators analyzed 20 randomized controlled trials of at least 50 patients and acceptable risk of bias in which VTC was used either for main or adjunct care delivery. Published from 2013 to 2019, these studies looked at care for diabetes and pain management, as well as some respiratory, neurologic, and cardiovascular conditions. Studies comparing VTC with usual care that did not involve any added in-person care were more likely to favor the VTC group, the investigators found.

“We excluded conditions such as substance use disorders, maternal care, and weight management for which there was sufficient prior evidence of the benefit of VTC,” Dr. Albritton said in an interview. “But I don’t think our results would have been substantially different if we had included these other diseases. We found general evidence in the literature that VTC is effective for a broader range of conditions.”

In some cases, such as if changes in a patient’s condition triggered an automatic virtual visit, the author said he thinks VTC may lead to even greater effectiveness.

“The doctor and patient could figure out on the spot what’s going on and perhaps change the medication,” Dr. Albritton explained.

In general agreement is Julia L. Frydman, MD, assistant professor in the Brookdale Department of Geriatric and Palliative Medicine at Icahn School of Medicine at Mount Sinai in New York, who was not involved in the RTI research.

Julia L. Frydman, MD, Icahn School of Medicine at Mount Sinai in New York
Dr. Julia L. Frydman

“Telemedicine has promise across many medical subspecialties, and what we need now are more studies to understand the perspectives of patients, caregivers, and clinicians as well as the impact of telemedicine on health outcomes and healthcare utilization.”

In acknowledgment of their utility, video visits are on the rise in the United States. A 2020 survey found that 22% of patients and 80% of physicians reported having participated in a video visit, three times the rate of the previous year. The authors noted that policy changes enacted to support telehealth strategies during the pandemic are expected to remain in place, and although patients are returning to in-person care, the virtual visit market will likely continue growing.
 

 

 

Increased telemedicine use by older adults

“We’ve seen an exciting expansion of telemedicine use among older adults, and we need to focus on continuing to meet their needs,” Dr. Frydman said.

In a recent study of televisits during the pandemic, Dr. Frydman’s group found a fivefold greater uptake of remote consultations by seniors – from 5% to 25%. Although in-person visits were far more common among older adults.

A specific advantage of video-based over audio-only telehealth, noted Dr. Albritton, is that physicians can directly observe patients in their home environment. Sharing that view is Deepa Iyengar, MBBS/MD,MPH, professor of family medicine at McGovern Medical School at The University of Texas Health Science Center at Houston, where, she said, “the pandemic has put VTC use into overdrive.”

Deepa Iyengar, MBBS/MD ,MPH, University of Texas Health Science Center at Houston
Dr. Deepa Iyengar

According to Dr. Iyengar, who was not involved in the RTI research, the video component definitely represents value-added over phone calls. “You can pick up visual cues on video that you might not see if the patient came in and you can see what the home environment is like – whether there are a lot of loose rugs on the floor or broken or missing light bulbs,” she said in an interview.
 

‘VTC is here to stay’

In other parts of the country, doctors are finding virtual care useful – and more common. “VTC is here to stay, for sure – the horse is out of the barn,” said Cheryl L. Wilkes, MD, an internist at Northwestern Medicine and assistant professor of medicine at Northwestern University in Chicago. “The RTI study shows no harm from VTC and also shows it may even improve clinical outcomes.”

Cheryl L. Wilkes, MD, Northwestern University, Chicago
Dr. Cheryl L. Wilkes

Video visits can also save patients high parking fees at clinics and spare the sick or elderly from having to hire caregivers to bring them into the office or from having to walk blocks in dangerous weather conditions, she added. “And I can do a virtual visit on the fly or at night when a relative or caregiver is home from work to be there with the patient.”

In addition to being beneficial for following up with patients with chronic diseases such as hypertension or diabetes, VTC may be able to replace some visits that have traditionally required hands-on care, said Dr. Wilkes.

She said she knows a cardiologist who has refined a process whereby a patient – say, one who may have edema – is asked to perform a maneuver via VTC and then display the result to the doctor: The doctor says, “put your leg up and press on it hard for 10 seconds and then show me what it looks like,” according to Dr. Wilkes.

The key now is to identify the best persons across specialties from neurology to rheumatology to videotape ways they’ve created to help their patients participate virtually in consults traditionally done at the office, Dr. Wilkes noted.

But some conditions will always require palpation and the use of a stethoscope, according Dr. Iyengar.

“If someone has an ulcer, I have to be able to feel it,” she said.

And while some maternity care can be given virtually – for instance, if a mother-to be develops a bad cold – hands-on obstetrical care to check the position and health of the baby obviously has to be done in person. “So VTC is definitely going to be a welcome addition but not a replacement,” Dr. Iyengar said.

Gaps in research on VTC visits

Many questions remain regarding the overall usefulness of VTC visits for certain patient groups, according to the authors.

They highlighted, for example, the dearth of data on subgroups or on underserved and vulnerable populations, with no head-to-head studies identified in their review. In addition, they found no studies examining VTC versus usual care for patients with concurrent conditions or on its effect on health equity and disparities.

“It’s now our job to understand the ongoing barriers to telemedicine access, including the digital divide and the usability of telemedicine platforms, and design interventions that overcome them,” Dr. Frydman said. “At the same time, we need to make sure we’re understanding and respecting the preferences of older adults in terms of how they access health care.”

This study was supported by the Patient-Centered Outcomes Research Institute (PCORI). Dr. Albritton is employed by RTI International, the contractor responsible for conducting the research and developing the manuscript. Several coauthors disclosed support from or contracts with PCORI. One coauthor’s spouse holds stock in private health companies. Dr. Frydman, Dr. Iyengar, and Dr. Wilkes disclosed no competing interests relevant to their comments.

Publications
Publications
Topics
Article Type
Sections
Article Source

FROM ANNALS OF INTERNAL MEDICINE

Disallow All Ads
Content Gating
No Gating (article Unlocked/Free)
Alternative CME
Disqus Comments
Default
Use ProPublica
Hide sidebar & use full width
render the right sidebar.
Conference Recap Checkbox
Not Conference Recap
Clinical Edge
Display the Slideshow in this Article
Medscape Article
Display survey writer
Reuters content
Disable Inline Native ads
WebMD Article

Hemostatic powder noninferior in nonvariceal upper GI bleeds

Article Type
Changed
Tue, 12/07/2021 - 12:40

TC-325, a bentonite-derived hemostatic powder, was not inferior to standard therapy for the endoscopic management of acute nonvariceal upper GI bleeding, according to a new study.

The findings from the study, lead investigator and study author James Y.W. Lau, MD, of the Prince of Wales Hospital in Hong Kong, said in an interview, suggest TC-325 could “be considered one of the primary endoscopic treatments to actively stop nonvariceal bleeding,” particularly in cases when other therapies prove unsuccessful. The study findings were published in Annals of Internal Medicine.

The study team noted that, after they first reported the use of TC-325 in active bleeding from gastroduodenal ulcers in 2011, there have been other studies of its use with acute nonvariceal upper GI bleeding, but to date there has been only two randomized controlled trials of it as a sole endoscopic treatment option for acute nonvariceal upper GI bleeding. To close this research gap, Dr. Lau and researchers enrolled 224 adult patients with acute bleeding from a nonvariceal source on upper GI endoscopy and randomly assigned these patients to receive either TC-325 (n = 111) or standard hemostatic treatment (n = 113). Standard endoscopic bleeding management consisted of contact thermocoagulation using a heater probe or bipolar probe, or hemoclipping with or without previously injected diluted epinephrine.

Success of assigned treatment was defined by the cessation of active bleeding as well as flattening of the protuberance or vessel with a heater or bipolar probe. For the primary outcome of the study, the investigators assessed the rate of bleeding control within 30 days following randomization. Additionally, the researchers compared the treatment groups to identify differences in the failure to control bleeding during the initial endoscopy and recurrent bleeding following hemostasis.

Treatment groups were even in regard to the proportions of patients with bleeding gastroduodenal ulcers (61.3% vs. 60.2%). A smaller proportion of patients in the TC-325 arm had a history of alcohol use (3.0% vs. 9.8%) and current use of NSAIDs (8.1% vs. 20.4%). The group assigned to TC-325 had more bleeding tumors (20.7% vs. 8.8%) and fewer Dieulafoy lesions (5.4% vs. 14.2%), compared with the standard treatment arm. Additionally, patients in the TC-325 group had a higher median Glasgow-Blatchford Score at hospital admission than the standard endoscopy management group (12 vs. 11, respectively; P < .05).

Although a greater proportion of patients assigned TC-325 had bleeding controlled within 30 days of randomization (90.1% vs. 81.4%; risk difference, 8.7 percentage points; 1-sided 95% CI, 0.95 percentage points), the researchers noted that the lower limit of the confidence interval for treatment difference “did not extend beyond the prespecified noninferiority margin of 10 percentage points, indicating that TC-325 is not inferior to standard treatment in the control of bleeding.”

Fewer failures of hemostasis were observed with TC-325 during index endoscopy (2.7% vs. 9.7%; odds ratio, 0.26; 95% CI, 0.07-0.95). After initial endoscopic control, recurrent bleeding was observed in 9 patients in the TC-325 arm and 10 patients in the standard treatment group.

The authors suggested that the low recurrent bleeding rate in the TC-325 arm may reflect enhanced responsiveness in the predominantly Asian study population, a group with lower parietal cell masses and higher rates of Helicobacter pylori infections. In an accompanying editorial published online in Annals of Internal Medicine, Alan N. Barkun, MD, McGill University and McGill University Health Centre in Montreal, and Ali Alali, MB BCh BAO, in the department of medicine at Kuwait University, Kuwait City, noted that “possible additional reasons for the enhanced effectiveness of TC-325 observed in the current trial may be its varied performance in the patients with nonulcer bleeding.”

No difference was found between the treatment strategies in terms of the need for additional interventions within 30 days. The need for further endoscopic treatment was reported in 7.2% of patients in the TC-325 groups versus 8.8% of patients assigned to standard treatment. In addition, further angiography was required in 1.8% and 3.5% of patients, while further surgery was required in 0.9% of patients treated with TC-325 versus none in the standard treatment group. Each group reported 14 deaths.

Dr. Lau noted that the study enrolled Asian patients who were more responsive to proton pump inhibitor therapy, which may limit the generalizability of the findings. “We also included patients with mixed etiologies,” he added. “Studies that focus on specific lesions would further inform our practice, and larger observational studies are required to understand failures with TC-325.”

Based on the study findings, corresponding editorial author Dr. Barkun wrote that “TC-325 can be used alone in nonvariceal upper gastrointestinal bleeding or as rescue therapy but should be reserved for patients with actively bleeding lesions” and suggests the treatment option “is likely one of the most effective modalities in achieving immediate hemostasis.”

The researchers reported no conflicts of interest with the pharmaceutical industry. The editorialists also reported no disclosures of interest.

Publications
Topics
Sections

TC-325, a bentonite-derived hemostatic powder, was not inferior to standard therapy for the endoscopic management of acute nonvariceal upper GI bleeding, according to a new study.

The findings from the study, lead investigator and study author James Y.W. Lau, MD, of the Prince of Wales Hospital in Hong Kong, said in an interview, suggest TC-325 could “be considered one of the primary endoscopic treatments to actively stop nonvariceal bleeding,” particularly in cases when other therapies prove unsuccessful. The study findings were published in Annals of Internal Medicine.

The study team noted that, after they first reported the use of TC-325 in active bleeding from gastroduodenal ulcers in 2011, there have been other studies of its use with acute nonvariceal upper GI bleeding, but to date there has been only two randomized controlled trials of it as a sole endoscopic treatment option for acute nonvariceal upper GI bleeding. To close this research gap, Dr. Lau and researchers enrolled 224 adult patients with acute bleeding from a nonvariceal source on upper GI endoscopy and randomly assigned these patients to receive either TC-325 (n = 111) or standard hemostatic treatment (n = 113). Standard endoscopic bleeding management consisted of contact thermocoagulation using a heater probe or bipolar probe, or hemoclipping with or without previously injected diluted epinephrine.

Success of assigned treatment was defined by the cessation of active bleeding as well as flattening of the protuberance or vessel with a heater or bipolar probe. For the primary outcome of the study, the investigators assessed the rate of bleeding control within 30 days following randomization. Additionally, the researchers compared the treatment groups to identify differences in the failure to control bleeding during the initial endoscopy and recurrent bleeding following hemostasis.

Treatment groups were even in regard to the proportions of patients with bleeding gastroduodenal ulcers (61.3% vs. 60.2%). A smaller proportion of patients in the TC-325 arm had a history of alcohol use (3.0% vs. 9.8%) and current use of NSAIDs (8.1% vs. 20.4%). The group assigned to TC-325 had more bleeding tumors (20.7% vs. 8.8%) and fewer Dieulafoy lesions (5.4% vs. 14.2%), compared with the standard treatment arm. Additionally, patients in the TC-325 group had a higher median Glasgow-Blatchford Score at hospital admission than the standard endoscopy management group (12 vs. 11, respectively; P < .05).

Although a greater proportion of patients assigned TC-325 had bleeding controlled within 30 days of randomization (90.1% vs. 81.4%; risk difference, 8.7 percentage points; 1-sided 95% CI, 0.95 percentage points), the researchers noted that the lower limit of the confidence interval for treatment difference “did not extend beyond the prespecified noninferiority margin of 10 percentage points, indicating that TC-325 is not inferior to standard treatment in the control of bleeding.”

Fewer failures of hemostasis were observed with TC-325 during index endoscopy (2.7% vs. 9.7%; odds ratio, 0.26; 95% CI, 0.07-0.95). After initial endoscopic control, recurrent bleeding was observed in 9 patients in the TC-325 arm and 10 patients in the standard treatment group.

The authors suggested that the low recurrent bleeding rate in the TC-325 arm may reflect enhanced responsiveness in the predominantly Asian study population, a group with lower parietal cell masses and higher rates of Helicobacter pylori infections. In an accompanying editorial published online in Annals of Internal Medicine, Alan N. Barkun, MD, McGill University and McGill University Health Centre in Montreal, and Ali Alali, MB BCh BAO, in the department of medicine at Kuwait University, Kuwait City, noted that “possible additional reasons for the enhanced effectiveness of TC-325 observed in the current trial may be its varied performance in the patients with nonulcer bleeding.”

No difference was found between the treatment strategies in terms of the need for additional interventions within 30 days. The need for further endoscopic treatment was reported in 7.2% of patients in the TC-325 groups versus 8.8% of patients assigned to standard treatment. In addition, further angiography was required in 1.8% and 3.5% of patients, while further surgery was required in 0.9% of patients treated with TC-325 versus none in the standard treatment group. Each group reported 14 deaths.

Dr. Lau noted that the study enrolled Asian patients who were more responsive to proton pump inhibitor therapy, which may limit the generalizability of the findings. “We also included patients with mixed etiologies,” he added. “Studies that focus on specific lesions would further inform our practice, and larger observational studies are required to understand failures with TC-325.”

Based on the study findings, corresponding editorial author Dr. Barkun wrote that “TC-325 can be used alone in nonvariceal upper gastrointestinal bleeding or as rescue therapy but should be reserved for patients with actively bleeding lesions” and suggests the treatment option “is likely one of the most effective modalities in achieving immediate hemostasis.”

The researchers reported no conflicts of interest with the pharmaceutical industry. The editorialists also reported no disclosures of interest.

TC-325, a bentonite-derived hemostatic powder, was not inferior to standard therapy for the endoscopic management of acute nonvariceal upper GI bleeding, according to a new study.

The findings from the study, lead investigator and study author James Y.W. Lau, MD, of the Prince of Wales Hospital in Hong Kong, said in an interview, suggest TC-325 could “be considered one of the primary endoscopic treatments to actively stop nonvariceal bleeding,” particularly in cases when other therapies prove unsuccessful. The study findings were published in Annals of Internal Medicine.

The study team noted that, after they first reported the use of TC-325 in active bleeding from gastroduodenal ulcers in 2011, there have been other studies of its use with acute nonvariceal upper GI bleeding, but to date there has been only two randomized controlled trials of it as a sole endoscopic treatment option for acute nonvariceal upper GI bleeding. To close this research gap, Dr. Lau and researchers enrolled 224 adult patients with acute bleeding from a nonvariceal source on upper GI endoscopy and randomly assigned these patients to receive either TC-325 (n = 111) or standard hemostatic treatment (n = 113). Standard endoscopic bleeding management consisted of contact thermocoagulation using a heater probe or bipolar probe, or hemoclipping with or without previously injected diluted epinephrine.

Success of assigned treatment was defined by the cessation of active bleeding as well as flattening of the protuberance or vessel with a heater or bipolar probe. For the primary outcome of the study, the investigators assessed the rate of bleeding control within 30 days following randomization. Additionally, the researchers compared the treatment groups to identify differences in the failure to control bleeding during the initial endoscopy and recurrent bleeding following hemostasis.

Treatment groups were even in regard to the proportions of patients with bleeding gastroduodenal ulcers (61.3% vs. 60.2%). A smaller proportion of patients in the TC-325 arm had a history of alcohol use (3.0% vs. 9.8%) and current use of NSAIDs (8.1% vs. 20.4%). The group assigned to TC-325 had more bleeding tumors (20.7% vs. 8.8%) and fewer Dieulafoy lesions (5.4% vs. 14.2%), compared with the standard treatment arm. Additionally, patients in the TC-325 group had a higher median Glasgow-Blatchford Score at hospital admission than the standard endoscopy management group (12 vs. 11, respectively; P < .05).

Although a greater proportion of patients assigned TC-325 had bleeding controlled within 30 days of randomization (90.1% vs. 81.4%; risk difference, 8.7 percentage points; 1-sided 95% CI, 0.95 percentage points), the researchers noted that the lower limit of the confidence interval for treatment difference “did not extend beyond the prespecified noninferiority margin of 10 percentage points, indicating that TC-325 is not inferior to standard treatment in the control of bleeding.”

Fewer failures of hemostasis were observed with TC-325 during index endoscopy (2.7% vs. 9.7%; odds ratio, 0.26; 95% CI, 0.07-0.95). After initial endoscopic control, recurrent bleeding was observed in 9 patients in the TC-325 arm and 10 patients in the standard treatment group.

The authors suggested that the low recurrent bleeding rate in the TC-325 arm may reflect enhanced responsiveness in the predominantly Asian study population, a group with lower parietal cell masses and higher rates of Helicobacter pylori infections. In an accompanying editorial published online in Annals of Internal Medicine, Alan N. Barkun, MD, McGill University and McGill University Health Centre in Montreal, and Ali Alali, MB BCh BAO, in the department of medicine at Kuwait University, Kuwait City, noted that “possible additional reasons for the enhanced effectiveness of TC-325 observed in the current trial may be its varied performance in the patients with nonulcer bleeding.”

No difference was found between the treatment strategies in terms of the need for additional interventions within 30 days. The need for further endoscopic treatment was reported in 7.2% of patients in the TC-325 groups versus 8.8% of patients assigned to standard treatment. In addition, further angiography was required in 1.8% and 3.5% of patients, while further surgery was required in 0.9% of patients treated with TC-325 versus none in the standard treatment group. Each group reported 14 deaths.

Dr. Lau noted that the study enrolled Asian patients who were more responsive to proton pump inhibitor therapy, which may limit the generalizability of the findings. “We also included patients with mixed etiologies,” he added. “Studies that focus on specific lesions would further inform our practice, and larger observational studies are required to understand failures with TC-325.”

Based on the study findings, corresponding editorial author Dr. Barkun wrote that “TC-325 can be used alone in nonvariceal upper gastrointestinal bleeding or as rescue therapy but should be reserved for patients with actively bleeding lesions” and suggests the treatment option “is likely one of the most effective modalities in achieving immediate hemostasis.”

The researchers reported no conflicts of interest with the pharmaceutical industry. The editorialists also reported no disclosures of interest.

Publications
Publications
Topics
Article Type
Sections
Article Source

FROM ANNALS OF INTERNAL MEDICINE

Disallow All Ads
Content Gating
No Gating (article Unlocked/Free)
Alternative CME
Disqus Comments
Default
Use ProPublica
Hide sidebar & use full width
render the right sidebar.
Conference Recap Checkbox
Not Conference Recap
Clinical Edge
Display the Slideshow in this Article
Medscape Article
Display survey writer
Reuters content
Disable Inline Native ads
WebMD Article

Proper Use and Compliance of Facial Masks During the COVID-19 Pandemic: An Observational Study of Hospitals in New York City

Article Type
Changed
Tue, 12/07/2021 - 10:43
Display Headline
Proper Use and Compliance of Facial Masks During the COVID-19 Pandemic: An Observational Study of Hospitals in New York City

Although the universal use of masks by both health care professionals and the general public now appears routine, widely differing recommendations were distributed by different health organizations early in the pandemic. In April 2020, the World Health Organization (WHO) stated that there was no evidence that healthy individuals wearing a medical mask in the community prevented COVID-19 infection.1 However, these recommendations must be placed in the context of a national shortage of personal protective equipment early in the pandemic. The WHO guidance released on June 5, 2020, recommended continuous use of masks for health care workers in the clinical setting.2 Additional recommendations included mask replacement when wet, soiled, or damaged, and when the wearer touched the mask. The WHO also recommended mask usage by those with underlying medical comorbidities and those living in high population–density areas and in settings where physical distancing was not possible.2

The Centers for Disease Control and Prevention (CDC) officially recommended the use of face coverings for the general public to prevent COVID-19 transmission on April 3, 2020.3 The CDC highlighted that masks should not be worn by children younger than 2 years; individuals with respiratory compromise; and patients who are unconscious, incapacitated, or unable to remove a mask without assistance.4 Medical masks and respirators were only recommended for health care workers. Importantly, masks with valves/vents were not recommended, as respiratory droplets can be emitted, defeating the purpose of source control.4 New York State mandated mask usage in public places starting on April 15, 2020.

These recommendations were based on the hypothesis that COVID-19 transmission occurs primarily via droplets and contact. In reality, SARS-CoV-2 transmission more likely occurs in a continuum from larger droplets to miniscule aerosols expelled from an infected person when talking, coughing, or sneezing.5,6 It should be noted that there was a formal suggestion of the potential for airborne transmission of SARS-CoV-2 by the CDC in a statement on September 18, 2020, that was subsequently retracted 3 days later.7,8 The CDC, reversing their prior recommendations, updated their guidance on October 5, 2020, endorsing prior reports that SARS-CoV-2 can be spread through aerosol transmission.8

Mask usage helps prevent viral spread by all individuals, especially those who are presymptomatic and asymptomatic. Presymptomatic individuals account for approximately 40% to 60% of transmissions, and asymptomatic individuals account for approximately 4% to 30% of infections by some models, which suggest these individuals are the drivers of the pandemic, more so than symptomatic individuals.9-15 Additionally, masking also may in effect reduce the amount of SARS-CoV-2 to which individuals are being exposed in the community.14 Universal masking is a relatively low-cost, low-risk intervention that may provide moderate benefit to the individual but substantial benefit to communities at large.10-13 Universal masking in other countries also has clearly demonstrated major benefits during the pandemic. Implementation of universal masking in Taiwan resulted in only approximately 440 COVID-19 cases and less than 10 deaths, despite a population of 23 million.16 South Korea, having experience with Middle East respiratory syndrome, also was able to quickly institute a mask policy for its citizens, resulting in approximately 94% compliance.17 Moreover, several mathematical models have shown that even imperfect use of masks on a population level can prevent disease transmission and should be instituted.18

Given the importance and potential benefits of mask usage, we investigated compliance and proper utilization of facial masks in New York City (NYC), once the epicenter of the pandemic in the United States. New York City and the rest of New York State experienced more than 1.13 million and 1.46 million cases of COVID-19, respectively, as of early November 2021.19 Nationwide, NYC had the greatest absolute death count of more than 34,634 and the greatest rate of death per 100,000 individuals of 412. In contrast, New York State, excluding NYC, had an absolute death count of more than 21,646 and a death rate per 100,000 individuals of 195 as of early November 2021.19 Now entering 20 months since the first case of COVID-19 in NYC, it continues to be vital for facial mask protocols to be emphasized as part of a comprehensive infection prevention protocol, especially in light of continued vaccine resistance, to help stall continued spread of SARS-CoV-2.20

We seek to show that despite months of policies for universal masking in NYC, there is still considerable mask noncompliance by the general public in health care settings where the use of masks is particularly imperative. We conducted an observational study investigating proper use of face masks of adults entering the main entrance of 4 hospitals located in NYC.

Methods

We observed mask usage in adults entering 4 hospitals in September 2020 (postsurge in NYC and prior to the availability of COVID-19 vaccinations). Hospitals were chosen to represent several types of health care delivery systems available in the United States and included a city, state, federal, and private hospital. Data collection was completed during peak traffic hours (8:00 am to 12:00 pm) on a weekday and continued until a total of 100 unique patients were observed at each site. Each hospital entrance was barricaded, and hospital staff were stationed at these entry points to take each individual’s temperature, screen for symptoms and exposure risk, verify patients’ appointments, and ensure proper mask wearing (in optimal circumstances). Data collectors (J.L. and N.M.) were stationed just past the barricade of each hospital’s entrance and observed those who entered. Individuals were not approached about the study, demographics, or the use and/or views about usage of facial masks. Children and hospital employees were excluded from data collection, with the exception of 1 hospital with a dedicated employee entrance where employees were observed for mask compliance. Except for vented/valved masks or makeshift masks fashioned out of scarfs, bandanas, or similar materials, the type of mask an individual wore was not distinguished (medical masks, cotton masks, or respirator-type masks were not differentiated).

 

 

Mask usage was observed and classified into several categories: correctly fitting mask over the nose and mouth, no face mask, mask usage with nose exposed, mask usage with mouth exposed, mask usage with both nose and mouth exposed (ie, mask on the chin/neck area), loosely fitting mask, vented/valved mask, or other form of face covering (eg, bandana, scarf).

Results

We observed a consistent rate of mask compliance between 72% and 85%, with an average of 78% of the 600 individuals observed wearing correctly fitting masks across the 4 hospitals included in this study (Table). The employee entrance included in this study had the highest compliance rate of 85%. An overall low rate of complete mask noncompliance was observed, with only 9 individuals (1.5%) in the entire study not wearing any mask. The federal hospital had the highest rate of mask noncompliance. We also observed a low rate of nose and mouth exposure, with 1.8% of individuals wearing a mask with the nose and mouth exposed (ie, mask tucked under the chin). No individuals were observed with the mouth exposed but with the nose covered by a mask. Additionally, only 3 individuals (0.5%) wore a mask with a vent/valve. The most common way that masks were worn incorrectly was with the nose exposed, accounting for 9.5% of individuals observed. Overall, only 9 individuals (1.5%) wore a nontraditional face covering, with a bandana being the most commonly observed makeshift mask.

Signage regarding the requirement to wear masks and to social distance was universally instituted at all hospital entry points (both inside and outside the hospital) in this study. However, there were no illustrations demonstrating correct and incorrect forms of mask usage. All signage merely displayed a graphic of a facial mask noting the requirement to wear a mask prior to entering the building. Hospital staff also had face masks available for patients who failed to bring a mask or who wore an inappropriate mask (ie, vented/valved masks).

 

Comment

Mask Effectiveness—Masks reduce the spread of SARS-CoV-2 by preventing both droplets and potentially virus-bearing aerosols.6,21,22 It has been demonstrated that well-fitted cotton homemade masks and medical masks provide the most effective method of reducing droplet dispersion. Loosely fitted masks as well as bandana-style facial coverings minimally reduce small aerosolized droplets, and an uncovered mouth and nose can disperse particles at a distance much greater than 6 feet.22

Mask Compliance—We report an overall high compliance rate with mask wearing among individuals visiting a hospital; however, compliance was still imperfect. Overall, 78% of observed individuals wore a correctly fitting mask when entering a hospital, even with hospital staff positioned at entry points to ensure proper mask usage. With all the resources available at health care centers, we anticipated a much higher compliance rate for correctly fitting masks at hospital entrances. We hypothesize that given only 78% of individuals showed proper mask compliance in a setting with enforcement by health care personnel, the mask compliance rate in the larger community is likely much lower. It is imperative to enforce continued mask compliance in medical centers and other public areas given notable vaccine noncompliance in certain parts of the country.

 

 

Tools to Prevent Disease Transmission—Mask usage by the general public in NYC helped in its response to the COVID-19 pandemic. Yang et al23 demonstrated through mathematical modeling that mask usage in NYC was associated with a 6.6% reduction in transmission overall and a 20% decrease in transmission for individuals 65 years and older during the first month of the universal mask policy going into effect. The authors extrapolated these data during the NYC reopening and found that universal masking reduced transmission by approximately 9% to 11%, accounting for the increase in hours spent outside home quarantine. The authors also hypothesized that if universal masking was as effective in its reduction of transmission for everyone in NYC as it was for older adults, the potential reduction in transmission of SARS-CoV-2 could be as high as 28% to 32%.23

Temperature checks at entrance barricades were standard protocol during the observation period. Although the main purpose of this study was to investigate compliance with and proper use of facial masks in a health care setting, it should be mentioned that, although temperature checks were being done on almost every person entering a hospital, the uniformity and practicality of this intervention has not been backed by substantial evidence. Although many nontouch thermometers are intended to capture a forehead temperature for the most accurate reading, the authors will share that in their observation, medical personnel screening individuals at hospital entrances were observed checking temperatures at any easily accessible body part, such as the forearm, hand, or neck. Furthermore, it has been reported that only approximately 40% of individuals with COVID-19 present with a fever.24 Many hospitals, including the 4 that were included in this investigation, have formal protocols for patients presenting with a fever, especially those presenting to an ambulatory center. Patients are usually instructed to call ahead if they have a fever, and a decision regarding next steps will be discussed with a health care provider. In addition, 1 meta-analysis on the symptoms of COVID-19 suggested that approximately 12% of infected patients are asymptomatic, likely a conservative estimate.25 Although we do not suggest that hospitals stop temperature checks, consistent temperature checks in anatomic locations intended for the specific thermometer used must be employed. Alternatively, a thermographic camera system that could detect heat signatures may be a way to screen faster, only necessitating that those above a threshold be assessed further.

The results of this study suggest that much greater effort is being placed on these temperature checks than on other equally important components of the entrance health assessment. This initial encounter at hospital entrances should serve as an opportunity for education on proper choice and use of masks with clear instructions that masks should not be removed unless directed by a health care provider and in a designated area, such as an examination room. The COVID-19 pandemic in the United States is likely the first time an individual is wearing these types of masks. Reiterating when and how often a mask should be changed (eg, when wet or soiled), how a soiled mask is not an effective mask, how a used mask should be discarded, ways to prevent self-contamination (ie, proper donning and doffing), and the importance of other infection-prevention behaviors—hand hygiene; social distancing; avoidance of touching the eyes, nose, and mouth with unwashed hands; and regular disinfecting of surfaces—should be practiced.11,26-29 Extended use and reuse of masks also can result in transmission of infection.30

Throughout the pandemic, our personal experience is that some patients often overtly refuse to wear a mask, citing underlying respiratory issues. The implications of patients not wearing a mask in a medical office and endangering other patients and staff are beyond the scope of this analysis. We will, however, comment briefly on the evidence behind this common concern. Matuschek et al31 found substantial adverse changes in respiratory rate, oxygen saturation, and CO2 levels in patients with severe chronic obstructive pulmonary disease who were wearing N95 respirators during a 6-minute walk test. Another study by Chan et al32 showed that nonmedical masks in healthy older adults in the community setting had no impact on oxygen saturation. Ultimately, the most effective mask a patient can wear is a mask that will be worn consistently.32

Populations With Limited Access to Masks—The COVID-19 pandemic disproportionately impacted disadvantaged populations, both in socioeconomic status and minority status. A disproportionate number of COVID-19 hospitalizations and deaths occurred in lower-income and minority populations.10 In fact, Lamb et al33 reported that NYC neighborhoods with a larger proportion of uninsured individuals with limited access to health care and overall lower socioeconomic status had a higher rate of SARS-CoV-2 positivity. A retrospective study in Louisiana showed that Black individuals accounted for 77% of hospitalizations and 71% of deaths due to COVID-19 in a population where only 31% of individuals identified as Black.10 Chu et al6 even asserted that policies should be put into place to address equity issues for populations with limited access to masks. We agree that policies should be put into action to ensure that individuals lacking the means to obtain appropriate masks or unable to obtain an adequate supply of masks be provided this new necessity. It has been calculated that the impact of masks in reducing virus transmission would be greatest if mask availability to disadvantaged populations is ensured.18 We support a plan for masks to be covered by government-sponsored health plans.

 

 

Study Limitations—Several limitations exist in our study that should be discussed. Although the data collectors observed a large number of individuals, each hospital entrance was only observed for 1 half-day morning session. There may be variations in the number of people wearing a mask at different times of day and different days of the week with fluctuations in hospital traffic. Although data were collected at a variety of hospitals representing the diverse health care delivery models available in the United States, the NYC hospitals included in this study may have different resources available for infection-prevention strategies than hospitals across the country, given NYC’s unique population density and demographics.

Study Strengths—The generalizability of the study should be recognized. Data were collected by all major health care delivery models available in the United States—private, state, city, and federal hospital systems. This study can be easily replicated in other health care delivery systems to further investigate potential gaps in mask usage and infection prevention. Repeating this study in areas where a large portion of the population does not believe in the virus also will likely show lower levels of mask use.

Conclusion

As the country grapples with vaccine hesitancy and with the new variants of SARS-CoV-2, continued universal masking is still imperative. The effectiveness of universal masking has been demonstrated, and with the combination of vaccinations, we can be assured that the world will continue to emerge from the pandemic.

References
  1. World Health Organization. Advice on the use of masks in the context of COVID-19. Interim guidance (6 April 2020). Accessed November 8, 2021. https://apps.who.int/iris/bitstream/handle/10665/331693/WHO-2019-nCov-IPC_Masks-2020.3-eng.pdf?sequence=1ceisAllowed=y
  2. World Health Organization. Advice on the use of masks in the context of COVID-19. Interim guidance (5 June 2020). Accessed November 8, 2021. https://apps.who.int/iris/bitstream/handle/10665/332293/WHO- 2019-nCov-IPC_Masks-2020.4-eng.pdf?sequence=1&isAllowed=y
  3. Fisher KA, Barile JP, Guerin RJ, et al. Factors associated with cloth face covering use among adults during the COVID-19 pandemic—United States, April and May 2020. MMWR Morb Mortal Wkly Rep. 2020;69:933-937.
  4. Centers for Disease Control and Prevention. Coronavirus disease 2019 (COVID-19). Considerations for wearing masks (19 April 2021). Accessed November 10, 2021. https://www.cdc.gov/coronavirus/2019-ncov/prevent-getting-sick/cloth-face-cover-guidance.html
  5. Conly J, Seto WH, Pittet D, et al. Use of medical face masks versus particulate respirators as a component of personal protective equipment for health care workers in the context of the COVID-19 pandemic. Antimicrob Resist Infect Control. 2020;9:126. 
  6. Chu DK, Akl EA, Duda S, et al; COVID-19 Systematic Urgent Review Group Effort (SURGE) study authors. Physical distancing, face masks, and eye protection to prevent person-to-person transmission of SARS-CoV-2 and COVID-19: a systematic review and meta-analysis. Lancet. 2020;395:1973-1987.
  7. Huang, P. Coronavirus FAQs: Why can’t the CDC make up its mind about airborne transmission? NPR. September 25, 2020. Accessed November 8, 2021. https://www.npr.org/sections/goatsandsoda/2020/09/25/916624967/coronavirus-faqs-why-cant-the-cdc-make-up-its-mind-about-airborne-transmission
  8. Centers for Disease Control and Prevention. Coronavirus disease 2019 (COVID-19). How COVID-19 spreads (14 July 2021). Accessed November 10, 2021. https://www.cdc.gov/coronavirus/2019-ncov/prevent-getting-sick/how-covid-spreads.html
  9. Wiersinga WJ, Rhodes A, Cheng AC, et al. Pathophysiology, transmission, diagnosis, and treatment of coronavirus disease 2019 (COVID-19): a review. JAMA. 2020;324:782-793. 
  10. Klompas M, Morris CA, Shenoy ES. Universal masking in the covid-19 era. N Engl J Med. 2020;383:E9.
  11. Middleton JD, Lopes H. Face masks in the covid-19 crisis: caveats, limits, and priorities. BMJ. 2020;369:m2030.
  12. Cheng KK, Lam TH, Leung CC. Wearing face masks in the community during the COVID-19 pandemic: altruism and solidarity [published online April 16, 2020]. Lancet. doi:10.1016/S0140-6736(20)30918-1
  13. Javid B, Weekes MP, Matheson NJ. Covid-19: should the public wear face masks? BMJ. 2020;369:m1442.
  14. Gandhi M, Beyrer C, Goosby E. Masks do more than protect others during COVID-19: reducing the inoculum of SARS-CoV-2 to protect the wearer. J Gen Intern Med. 2020;35:3063-3066.
  15. Ngonghala CN, Iboi EA, Gumel AB. Could masks curtail the post-lockdown resurgence of COVID-19 in the US? Math Biosci. 2020;329:108452. doi:10.1016/j.mbs.2020.108452
  16. Yi-Fong Su V, Yen YF, Yang KY, et al. Masks and medical care: two keys to Taiwan’s success in preventing COVID-19 spread. Travel Med Infect Dis. 2020;38:101780.
  17. Lim S, Yoon HI, Song KH, et al. Face masks and containment of COVID-19: experience from South Korea. J Hosp Infect. 2020;106:206-207.
  18. Fisman DN, Greer AL, Tuite AR. Bidirectional impact of imperfect mask use on reproduction number of COVID-19: a next generation matrix approach. Infect Dis Model. 2020;5:405-408.
  19. Centers for Disease Control and Prevention. COVID data tracker. United States COVID-19 cases, deaths, and laboratory testing (NAATs) by state, territory, and jurisdiction. Accessed July 6, 2021. https://covid.cdc.gov/covid-data-tracker/#cases_totalcases
  20. Francescani C. Timeline: the first 100 days of New York Gov. Andrew Cuomo’s COVID-19 response. ABC News. June 17, 2020. Accessed November 8, 2021. https://abcnews.go.com/US/News/timeline-100-days-york-gov-andrew-cuomos-covid/story?id=71292880
  21. Zhang R, Li Y, Zhang AL, et al. Identifying airborne transmission as the dominant route for the spread of COVID-19. Proc Natl Acad Sci U S A. 2020;117:14857-14863. 
  22. Verma S, Dhanak M, Frankenfield J. Visualizing the effectiveness of face masks in obstructing respiratory jets. Phys Fluids (1994). 2020;32:061708.
  23. Yang W, Shaff J, Shaman J. COVID-19 transmission dynamics and effectiveness of public health interventions in New York City during the 2020 spring pandemic wave. medRxiv. Preprint posted online September 9, 2020. doi:10.1101/2020.09.08.20190710
  24. Zavascki AP, Falci DR. Clinical characteristics of covid-19 in China. N Engl J Med. 2020;382:1859. 
  25. Zhu J, Ji P, Pang J, et al. Clinical characteristics of 3062 COVID-19 patients: a meta-analysis. J Med Virol. 2020;92:1902-1914. doi:10.1002/jmv.25884
  26. Sommerstein R, Fux CA, Vuichard-Gysin D, et al. Risk of SARS-CoV-2 transmission by aerosols, the rational use of masks, and protection of healthcare workers from COVID-19. Antimicrob Resist Infect Control. 2020;9:100.
  27. Stone TE, Kunaviktikul W, Omura M, et al. Facemasks and the covid 19 pandemic: what advice should health professionals be giving the general public about the wearing of facemasks? Nurs Health Sci. 2020;22:339-342.
  28. Tam VC, Tam SY, Poon WK, et al. A reality check on the use of face masks during the COVID-19 outbreak in Hong Kong. EClinicalMedicine. 2020;22:100356.
  29. Chen YJ, Qin G, Chen J, et al. Comparison of face-touching behaviors before and during the coronavirus disease 2019 pandemic. JAMA Netw Open. 2020;3:e2016924. 
  30. O’Dowd K, Nair KM, Forouzandeh P, et al. Face masks and respirators in the fight against the COVID-19 pandemic: a review of current materials, advances and future perspectives. Materials (Basel). 2020;13:3363.
  31. Matuschek C, Moll F, Fangerau H, et al. Face masks: benefits and risks during the COVID-19 crisis. Eur J Med Res. 2020;25:32.
  32. Chan NC, Li K, Hirsh J. Peripheral oxygen saturation in older persons wearing nonmedical face masks in community settings. JAMA. 2020;324:2323-2324. doi:10.1001/jama.2020.21905
  33. Lamb MRKandula SShaman JDifferential COVID‐19 case positivity in New York City neighborhoods: socioeconomic factors and mobilityInfluenza Other Respir Viruses2021;15:209-217. doi:10.1111/irv.12816
Article PDF
Author and Disclosure Information

From the Department of Dermatology, State University of New York Downstate Medical Center, Brooklyn, New York. Dr. Siegel also is from the Department of Dermatology, VA New York Harbor Healthcare System, Brooklyn, New York.

The authors report no conflict of interest.

Correspondence: Jameson Loyal, MD, Department of Dermatology, 450 Clarkson Ave, MSC 46, Brooklyn, NY 11203 (Jameson.Loyal@gmail.com).

Issue
Cutis - 108(6)
Publications
Topics
Page Number
333-337
Sections
Author and Disclosure Information

From the Department of Dermatology, State University of New York Downstate Medical Center, Brooklyn, New York. Dr. Siegel also is from the Department of Dermatology, VA New York Harbor Healthcare System, Brooklyn, New York.

The authors report no conflict of interest.

Correspondence: Jameson Loyal, MD, Department of Dermatology, 450 Clarkson Ave, MSC 46, Brooklyn, NY 11203 (Jameson.Loyal@gmail.com).

Author and Disclosure Information

From the Department of Dermatology, State University of New York Downstate Medical Center, Brooklyn, New York. Dr. Siegel also is from the Department of Dermatology, VA New York Harbor Healthcare System, Brooklyn, New York.

The authors report no conflict of interest.

Correspondence: Jameson Loyal, MD, Department of Dermatology, 450 Clarkson Ave, MSC 46, Brooklyn, NY 11203 (Jameson.Loyal@gmail.com).

Article PDF
Article PDF

Although the universal use of masks by both health care professionals and the general public now appears routine, widely differing recommendations were distributed by different health organizations early in the pandemic. In April 2020, the World Health Organization (WHO) stated that there was no evidence that healthy individuals wearing a medical mask in the community prevented COVID-19 infection.1 However, these recommendations must be placed in the context of a national shortage of personal protective equipment early in the pandemic. The WHO guidance released on June 5, 2020, recommended continuous use of masks for health care workers in the clinical setting.2 Additional recommendations included mask replacement when wet, soiled, or damaged, and when the wearer touched the mask. The WHO also recommended mask usage by those with underlying medical comorbidities and those living in high population–density areas and in settings where physical distancing was not possible.2

The Centers for Disease Control and Prevention (CDC) officially recommended the use of face coverings for the general public to prevent COVID-19 transmission on April 3, 2020.3 The CDC highlighted that masks should not be worn by children younger than 2 years; individuals with respiratory compromise; and patients who are unconscious, incapacitated, or unable to remove a mask without assistance.4 Medical masks and respirators were only recommended for health care workers. Importantly, masks with valves/vents were not recommended, as respiratory droplets can be emitted, defeating the purpose of source control.4 New York State mandated mask usage in public places starting on April 15, 2020.

These recommendations were based on the hypothesis that COVID-19 transmission occurs primarily via droplets and contact. In reality, SARS-CoV-2 transmission more likely occurs in a continuum from larger droplets to miniscule aerosols expelled from an infected person when talking, coughing, or sneezing.5,6 It should be noted that there was a formal suggestion of the potential for airborne transmission of SARS-CoV-2 by the CDC in a statement on September 18, 2020, that was subsequently retracted 3 days later.7,8 The CDC, reversing their prior recommendations, updated their guidance on October 5, 2020, endorsing prior reports that SARS-CoV-2 can be spread through aerosol transmission.8

Mask usage helps prevent viral spread by all individuals, especially those who are presymptomatic and asymptomatic. Presymptomatic individuals account for approximately 40% to 60% of transmissions, and asymptomatic individuals account for approximately 4% to 30% of infections by some models, which suggest these individuals are the drivers of the pandemic, more so than symptomatic individuals.9-15 Additionally, masking also may in effect reduce the amount of SARS-CoV-2 to which individuals are being exposed in the community.14 Universal masking is a relatively low-cost, low-risk intervention that may provide moderate benefit to the individual but substantial benefit to communities at large.10-13 Universal masking in other countries also has clearly demonstrated major benefits during the pandemic. Implementation of universal masking in Taiwan resulted in only approximately 440 COVID-19 cases and less than 10 deaths, despite a population of 23 million.16 South Korea, having experience with Middle East respiratory syndrome, also was able to quickly institute a mask policy for its citizens, resulting in approximately 94% compliance.17 Moreover, several mathematical models have shown that even imperfect use of masks on a population level can prevent disease transmission and should be instituted.18

Given the importance and potential benefits of mask usage, we investigated compliance and proper utilization of facial masks in New York City (NYC), once the epicenter of the pandemic in the United States. New York City and the rest of New York State experienced more than 1.13 million and 1.46 million cases of COVID-19, respectively, as of early November 2021.19 Nationwide, NYC had the greatest absolute death count of more than 34,634 and the greatest rate of death per 100,000 individuals of 412. In contrast, New York State, excluding NYC, had an absolute death count of more than 21,646 and a death rate per 100,000 individuals of 195 as of early November 2021.19 Now entering 20 months since the first case of COVID-19 in NYC, it continues to be vital for facial mask protocols to be emphasized as part of a comprehensive infection prevention protocol, especially in light of continued vaccine resistance, to help stall continued spread of SARS-CoV-2.20

We seek to show that despite months of policies for universal masking in NYC, there is still considerable mask noncompliance by the general public in health care settings where the use of masks is particularly imperative. We conducted an observational study investigating proper use of face masks of adults entering the main entrance of 4 hospitals located in NYC.

Methods

We observed mask usage in adults entering 4 hospitals in September 2020 (postsurge in NYC and prior to the availability of COVID-19 vaccinations). Hospitals were chosen to represent several types of health care delivery systems available in the United States and included a city, state, federal, and private hospital. Data collection was completed during peak traffic hours (8:00 am to 12:00 pm) on a weekday and continued until a total of 100 unique patients were observed at each site. Each hospital entrance was barricaded, and hospital staff were stationed at these entry points to take each individual’s temperature, screen for symptoms and exposure risk, verify patients’ appointments, and ensure proper mask wearing (in optimal circumstances). Data collectors (J.L. and N.M.) were stationed just past the barricade of each hospital’s entrance and observed those who entered. Individuals were not approached about the study, demographics, or the use and/or views about usage of facial masks. Children and hospital employees were excluded from data collection, with the exception of 1 hospital with a dedicated employee entrance where employees were observed for mask compliance. Except for vented/valved masks or makeshift masks fashioned out of scarfs, bandanas, or similar materials, the type of mask an individual wore was not distinguished (medical masks, cotton masks, or respirator-type masks were not differentiated).

 

 

Mask usage was observed and classified into several categories: correctly fitting mask over the nose and mouth, no face mask, mask usage with nose exposed, mask usage with mouth exposed, mask usage with both nose and mouth exposed (ie, mask on the chin/neck area), loosely fitting mask, vented/valved mask, or other form of face covering (eg, bandana, scarf).

Results

We observed a consistent rate of mask compliance between 72% and 85%, with an average of 78% of the 600 individuals observed wearing correctly fitting masks across the 4 hospitals included in this study (Table). The employee entrance included in this study had the highest compliance rate of 85%. An overall low rate of complete mask noncompliance was observed, with only 9 individuals (1.5%) in the entire study not wearing any mask. The federal hospital had the highest rate of mask noncompliance. We also observed a low rate of nose and mouth exposure, with 1.8% of individuals wearing a mask with the nose and mouth exposed (ie, mask tucked under the chin). No individuals were observed with the mouth exposed but with the nose covered by a mask. Additionally, only 3 individuals (0.5%) wore a mask with a vent/valve. The most common way that masks were worn incorrectly was with the nose exposed, accounting for 9.5% of individuals observed. Overall, only 9 individuals (1.5%) wore a nontraditional face covering, with a bandana being the most commonly observed makeshift mask.

Signage regarding the requirement to wear masks and to social distance was universally instituted at all hospital entry points (both inside and outside the hospital) in this study. However, there were no illustrations demonstrating correct and incorrect forms of mask usage. All signage merely displayed a graphic of a facial mask noting the requirement to wear a mask prior to entering the building. Hospital staff also had face masks available for patients who failed to bring a mask or who wore an inappropriate mask (ie, vented/valved masks).

 

Comment

Mask Effectiveness—Masks reduce the spread of SARS-CoV-2 by preventing both droplets and potentially virus-bearing aerosols.6,21,22 It has been demonstrated that well-fitted cotton homemade masks and medical masks provide the most effective method of reducing droplet dispersion. Loosely fitted masks as well as bandana-style facial coverings minimally reduce small aerosolized droplets, and an uncovered mouth and nose can disperse particles at a distance much greater than 6 feet.22

Mask Compliance—We report an overall high compliance rate with mask wearing among individuals visiting a hospital; however, compliance was still imperfect. Overall, 78% of observed individuals wore a correctly fitting mask when entering a hospital, even with hospital staff positioned at entry points to ensure proper mask usage. With all the resources available at health care centers, we anticipated a much higher compliance rate for correctly fitting masks at hospital entrances. We hypothesize that given only 78% of individuals showed proper mask compliance in a setting with enforcement by health care personnel, the mask compliance rate in the larger community is likely much lower. It is imperative to enforce continued mask compliance in medical centers and other public areas given notable vaccine noncompliance in certain parts of the country.

 

 

Tools to Prevent Disease Transmission—Mask usage by the general public in NYC helped in its response to the COVID-19 pandemic. Yang et al23 demonstrated through mathematical modeling that mask usage in NYC was associated with a 6.6% reduction in transmission overall and a 20% decrease in transmission for individuals 65 years and older during the first month of the universal mask policy going into effect. The authors extrapolated these data during the NYC reopening and found that universal masking reduced transmission by approximately 9% to 11%, accounting for the increase in hours spent outside home quarantine. The authors also hypothesized that if universal masking was as effective in its reduction of transmission for everyone in NYC as it was for older adults, the potential reduction in transmission of SARS-CoV-2 could be as high as 28% to 32%.23

Temperature checks at entrance barricades were standard protocol during the observation period. Although the main purpose of this study was to investigate compliance with and proper use of facial masks in a health care setting, it should be mentioned that, although temperature checks were being done on almost every person entering a hospital, the uniformity and practicality of this intervention has not been backed by substantial evidence. Although many nontouch thermometers are intended to capture a forehead temperature for the most accurate reading, the authors will share that in their observation, medical personnel screening individuals at hospital entrances were observed checking temperatures at any easily accessible body part, such as the forearm, hand, or neck. Furthermore, it has been reported that only approximately 40% of individuals with COVID-19 present with a fever.24 Many hospitals, including the 4 that were included in this investigation, have formal protocols for patients presenting with a fever, especially those presenting to an ambulatory center. Patients are usually instructed to call ahead if they have a fever, and a decision regarding next steps will be discussed with a health care provider. In addition, 1 meta-analysis on the symptoms of COVID-19 suggested that approximately 12% of infected patients are asymptomatic, likely a conservative estimate.25 Although we do not suggest that hospitals stop temperature checks, consistent temperature checks in anatomic locations intended for the specific thermometer used must be employed. Alternatively, a thermographic camera system that could detect heat signatures may be a way to screen faster, only necessitating that those above a threshold be assessed further.

The results of this study suggest that much greater effort is being placed on these temperature checks than on other equally important components of the entrance health assessment. This initial encounter at hospital entrances should serve as an opportunity for education on proper choice and use of masks with clear instructions that masks should not be removed unless directed by a health care provider and in a designated area, such as an examination room. The COVID-19 pandemic in the United States is likely the first time an individual is wearing these types of masks. Reiterating when and how often a mask should be changed (eg, when wet or soiled), how a soiled mask is not an effective mask, how a used mask should be discarded, ways to prevent self-contamination (ie, proper donning and doffing), and the importance of other infection-prevention behaviors—hand hygiene; social distancing; avoidance of touching the eyes, nose, and mouth with unwashed hands; and regular disinfecting of surfaces—should be practiced.11,26-29 Extended use and reuse of masks also can result in transmission of infection.30

Throughout the pandemic, our personal experience is that some patients often overtly refuse to wear a mask, citing underlying respiratory issues. The implications of patients not wearing a mask in a medical office and endangering other patients and staff are beyond the scope of this analysis. We will, however, comment briefly on the evidence behind this common concern. Matuschek et al31 found substantial adverse changes in respiratory rate, oxygen saturation, and CO2 levels in patients with severe chronic obstructive pulmonary disease who were wearing N95 respirators during a 6-minute walk test. Another study by Chan et al32 showed that nonmedical masks in healthy older adults in the community setting had no impact on oxygen saturation. Ultimately, the most effective mask a patient can wear is a mask that will be worn consistently.32

Populations With Limited Access to Masks—The COVID-19 pandemic disproportionately impacted disadvantaged populations, both in socioeconomic status and minority status. A disproportionate number of COVID-19 hospitalizations and deaths occurred in lower-income and minority populations.10 In fact, Lamb et al33 reported that NYC neighborhoods with a larger proportion of uninsured individuals with limited access to health care and overall lower socioeconomic status had a higher rate of SARS-CoV-2 positivity. A retrospective study in Louisiana showed that Black individuals accounted for 77% of hospitalizations and 71% of deaths due to COVID-19 in a population where only 31% of individuals identified as Black.10 Chu et al6 even asserted that policies should be put into place to address equity issues for populations with limited access to masks. We agree that policies should be put into action to ensure that individuals lacking the means to obtain appropriate masks or unable to obtain an adequate supply of masks be provided this new necessity. It has been calculated that the impact of masks in reducing virus transmission would be greatest if mask availability to disadvantaged populations is ensured.18 We support a plan for masks to be covered by government-sponsored health plans.

 

 

Study Limitations—Several limitations exist in our study that should be discussed. Although the data collectors observed a large number of individuals, each hospital entrance was only observed for 1 half-day morning session. There may be variations in the number of people wearing a mask at different times of day and different days of the week with fluctuations in hospital traffic. Although data were collected at a variety of hospitals representing the diverse health care delivery models available in the United States, the NYC hospitals included in this study may have different resources available for infection-prevention strategies than hospitals across the country, given NYC’s unique population density and demographics.

Study Strengths—The generalizability of the study should be recognized. Data were collected by all major health care delivery models available in the United States—private, state, city, and federal hospital systems. This study can be easily replicated in other health care delivery systems to further investigate potential gaps in mask usage and infection prevention. Repeating this study in areas where a large portion of the population does not believe in the virus also will likely show lower levels of mask use.

Conclusion

As the country grapples with vaccine hesitancy and with the new variants of SARS-CoV-2, continued universal masking is still imperative. The effectiveness of universal masking has been demonstrated, and with the combination of vaccinations, we can be assured that the world will continue to emerge from the pandemic.

Although the universal use of masks by both health care professionals and the general public now appears routine, widely differing recommendations were distributed by different health organizations early in the pandemic. In April 2020, the World Health Organization (WHO) stated that there was no evidence that healthy individuals wearing a medical mask in the community prevented COVID-19 infection.1 However, these recommendations must be placed in the context of a national shortage of personal protective equipment early in the pandemic. The WHO guidance released on June 5, 2020, recommended continuous use of masks for health care workers in the clinical setting.2 Additional recommendations included mask replacement when wet, soiled, or damaged, and when the wearer touched the mask. The WHO also recommended mask usage by those with underlying medical comorbidities and those living in high population–density areas and in settings where physical distancing was not possible.2

The Centers for Disease Control and Prevention (CDC) officially recommended the use of face coverings for the general public to prevent COVID-19 transmission on April 3, 2020.3 The CDC highlighted that masks should not be worn by children younger than 2 years; individuals with respiratory compromise; and patients who are unconscious, incapacitated, or unable to remove a mask without assistance.4 Medical masks and respirators were only recommended for health care workers. Importantly, masks with valves/vents were not recommended, as respiratory droplets can be emitted, defeating the purpose of source control.4 New York State mandated mask usage in public places starting on April 15, 2020.

These recommendations were based on the hypothesis that COVID-19 transmission occurs primarily via droplets and contact. In reality, SARS-CoV-2 transmission more likely occurs in a continuum from larger droplets to miniscule aerosols expelled from an infected person when talking, coughing, or sneezing.5,6 It should be noted that there was a formal suggestion of the potential for airborne transmission of SARS-CoV-2 by the CDC in a statement on September 18, 2020, that was subsequently retracted 3 days later.7,8 The CDC, reversing their prior recommendations, updated their guidance on October 5, 2020, endorsing prior reports that SARS-CoV-2 can be spread through aerosol transmission.8

Mask usage helps prevent viral spread by all individuals, especially those who are presymptomatic and asymptomatic. Presymptomatic individuals account for approximately 40% to 60% of transmissions, and asymptomatic individuals account for approximately 4% to 30% of infections by some models, which suggest these individuals are the drivers of the pandemic, more so than symptomatic individuals.9-15 Additionally, masking also may in effect reduce the amount of SARS-CoV-2 to which individuals are being exposed in the community.14 Universal masking is a relatively low-cost, low-risk intervention that may provide moderate benefit to the individual but substantial benefit to communities at large.10-13 Universal masking in other countries also has clearly demonstrated major benefits during the pandemic. Implementation of universal masking in Taiwan resulted in only approximately 440 COVID-19 cases and less than 10 deaths, despite a population of 23 million.16 South Korea, having experience with Middle East respiratory syndrome, also was able to quickly institute a mask policy for its citizens, resulting in approximately 94% compliance.17 Moreover, several mathematical models have shown that even imperfect use of masks on a population level can prevent disease transmission and should be instituted.18

Given the importance and potential benefits of mask usage, we investigated compliance and proper utilization of facial masks in New York City (NYC), once the epicenter of the pandemic in the United States. New York City and the rest of New York State experienced more than 1.13 million and 1.46 million cases of COVID-19, respectively, as of early November 2021.19 Nationwide, NYC had the greatest absolute death count of more than 34,634 and the greatest rate of death per 100,000 individuals of 412. In contrast, New York State, excluding NYC, had an absolute death count of more than 21,646 and a death rate per 100,000 individuals of 195 as of early November 2021.19 Now entering 20 months since the first case of COVID-19 in NYC, it continues to be vital for facial mask protocols to be emphasized as part of a comprehensive infection prevention protocol, especially in light of continued vaccine resistance, to help stall continued spread of SARS-CoV-2.20

We seek to show that despite months of policies for universal masking in NYC, there is still considerable mask noncompliance by the general public in health care settings where the use of masks is particularly imperative. We conducted an observational study investigating proper use of face masks of adults entering the main entrance of 4 hospitals located in NYC.

Methods

We observed mask usage in adults entering 4 hospitals in September 2020 (postsurge in NYC and prior to the availability of COVID-19 vaccinations). Hospitals were chosen to represent several types of health care delivery systems available in the United States and included a city, state, federal, and private hospital. Data collection was completed during peak traffic hours (8:00 am to 12:00 pm) on a weekday and continued until a total of 100 unique patients were observed at each site. Each hospital entrance was barricaded, and hospital staff were stationed at these entry points to take each individual’s temperature, screen for symptoms and exposure risk, verify patients’ appointments, and ensure proper mask wearing (in optimal circumstances). Data collectors (J.L. and N.M.) were stationed just past the barricade of each hospital’s entrance and observed those who entered. Individuals were not approached about the study, demographics, or the use and/or views about usage of facial masks. Children and hospital employees were excluded from data collection, with the exception of 1 hospital with a dedicated employee entrance where employees were observed for mask compliance. Except for vented/valved masks or makeshift masks fashioned out of scarfs, bandanas, or similar materials, the type of mask an individual wore was not distinguished (medical masks, cotton masks, or respirator-type masks were not differentiated).

 

 

Mask usage was observed and classified into several categories: correctly fitting mask over the nose and mouth, no face mask, mask usage with nose exposed, mask usage with mouth exposed, mask usage with both nose and mouth exposed (ie, mask on the chin/neck area), loosely fitting mask, vented/valved mask, or other form of face covering (eg, bandana, scarf).

Results

We observed a consistent rate of mask compliance between 72% and 85%, with an average of 78% of the 600 individuals observed wearing correctly fitting masks across the 4 hospitals included in this study (Table). The employee entrance included in this study had the highest compliance rate of 85%. An overall low rate of complete mask noncompliance was observed, with only 9 individuals (1.5%) in the entire study not wearing any mask. The federal hospital had the highest rate of mask noncompliance. We also observed a low rate of nose and mouth exposure, with 1.8% of individuals wearing a mask with the nose and mouth exposed (ie, mask tucked under the chin). No individuals were observed with the mouth exposed but with the nose covered by a mask. Additionally, only 3 individuals (0.5%) wore a mask with a vent/valve. The most common way that masks were worn incorrectly was with the nose exposed, accounting for 9.5% of individuals observed. Overall, only 9 individuals (1.5%) wore a nontraditional face covering, with a bandana being the most commonly observed makeshift mask.

Signage regarding the requirement to wear masks and to social distance was universally instituted at all hospital entry points (both inside and outside the hospital) in this study. However, there were no illustrations demonstrating correct and incorrect forms of mask usage. All signage merely displayed a graphic of a facial mask noting the requirement to wear a mask prior to entering the building. Hospital staff also had face masks available for patients who failed to bring a mask or who wore an inappropriate mask (ie, vented/valved masks).

 

Comment

Mask Effectiveness—Masks reduce the spread of SARS-CoV-2 by preventing both droplets and potentially virus-bearing aerosols.6,21,22 It has been demonstrated that well-fitted cotton homemade masks and medical masks provide the most effective method of reducing droplet dispersion. Loosely fitted masks as well as bandana-style facial coverings minimally reduce small aerosolized droplets, and an uncovered mouth and nose can disperse particles at a distance much greater than 6 feet.22

Mask Compliance—We report an overall high compliance rate with mask wearing among individuals visiting a hospital; however, compliance was still imperfect. Overall, 78% of observed individuals wore a correctly fitting mask when entering a hospital, even with hospital staff positioned at entry points to ensure proper mask usage. With all the resources available at health care centers, we anticipated a much higher compliance rate for correctly fitting masks at hospital entrances. We hypothesize that given only 78% of individuals showed proper mask compliance in a setting with enforcement by health care personnel, the mask compliance rate in the larger community is likely much lower. It is imperative to enforce continued mask compliance in medical centers and other public areas given notable vaccine noncompliance in certain parts of the country.

 

 

Tools to Prevent Disease Transmission—Mask usage by the general public in NYC helped in its response to the COVID-19 pandemic. Yang et al23 demonstrated through mathematical modeling that mask usage in NYC was associated with a 6.6% reduction in transmission overall and a 20% decrease in transmission for individuals 65 years and older during the first month of the universal mask policy going into effect. The authors extrapolated these data during the NYC reopening and found that universal masking reduced transmission by approximately 9% to 11%, accounting for the increase in hours spent outside home quarantine. The authors also hypothesized that if universal masking was as effective in its reduction of transmission for everyone in NYC as it was for older adults, the potential reduction in transmission of SARS-CoV-2 could be as high as 28% to 32%.23

Temperature checks at entrance barricades were standard protocol during the observation period. Although the main purpose of this study was to investigate compliance with and proper use of facial masks in a health care setting, it should be mentioned that, although temperature checks were being done on almost every person entering a hospital, the uniformity and practicality of this intervention has not been backed by substantial evidence. Although many nontouch thermometers are intended to capture a forehead temperature for the most accurate reading, the authors will share that in their observation, medical personnel screening individuals at hospital entrances were observed checking temperatures at any easily accessible body part, such as the forearm, hand, or neck. Furthermore, it has been reported that only approximately 40% of individuals with COVID-19 present with a fever.24 Many hospitals, including the 4 that were included in this investigation, have formal protocols for patients presenting with a fever, especially those presenting to an ambulatory center. Patients are usually instructed to call ahead if they have a fever, and a decision regarding next steps will be discussed with a health care provider. In addition, 1 meta-analysis on the symptoms of COVID-19 suggested that approximately 12% of infected patients are asymptomatic, likely a conservative estimate.25 Although we do not suggest that hospitals stop temperature checks, consistent temperature checks in anatomic locations intended for the specific thermometer used must be employed. Alternatively, a thermographic camera system that could detect heat signatures may be a way to screen faster, only necessitating that those above a threshold be assessed further.

The results of this study suggest that much greater effort is being placed on these temperature checks than on other equally important components of the entrance health assessment. This initial encounter at hospital entrances should serve as an opportunity for education on proper choice and use of masks with clear instructions that masks should not be removed unless directed by a health care provider and in a designated area, such as an examination room. The COVID-19 pandemic in the United States is likely the first time an individual is wearing these types of masks. Reiterating when and how often a mask should be changed (eg, when wet or soiled), how a soiled mask is not an effective mask, how a used mask should be discarded, ways to prevent self-contamination (ie, proper donning and doffing), and the importance of other infection-prevention behaviors—hand hygiene; social distancing; avoidance of touching the eyes, nose, and mouth with unwashed hands; and regular disinfecting of surfaces—should be practiced.11,26-29 Extended use and reuse of masks also can result in transmission of infection.30

Throughout the pandemic, our personal experience is that some patients often overtly refuse to wear a mask, citing underlying respiratory issues. The implications of patients not wearing a mask in a medical office and endangering other patients and staff are beyond the scope of this analysis. We will, however, comment briefly on the evidence behind this common concern. Matuschek et al31 found substantial adverse changes in respiratory rate, oxygen saturation, and CO2 levels in patients with severe chronic obstructive pulmonary disease who were wearing N95 respirators during a 6-minute walk test. Another study by Chan et al32 showed that nonmedical masks in healthy older adults in the community setting had no impact on oxygen saturation. Ultimately, the most effective mask a patient can wear is a mask that will be worn consistently.32

Populations With Limited Access to Masks—The COVID-19 pandemic disproportionately impacted disadvantaged populations, both in socioeconomic status and minority status. A disproportionate number of COVID-19 hospitalizations and deaths occurred in lower-income and minority populations.10 In fact, Lamb et al33 reported that NYC neighborhoods with a larger proportion of uninsured individuals with limited access to health care and overall lower socioeconomic status had a higher rate of SARS-CoV-2 positivity. A retrospective study in Louisiana showed that Black individuals accounted for 77% of hospitalizations and 71% of deaths due to COVID-19 in a population where only 31% of individuals identified as Black.10 Chu et al6 even asserted that policies should be put into place to address equity issues for populations with limited access to masks. We agree that policies should be put into action to ensure that individuals lacking the means to obtain appropriate masks or unable to obtain an adequate supply of masks be provided this new necessity. It has been calculated that the impact of masks in reducing virus transmission would be greatest if mask availability to disadvantaged populations is ensured.18 We support a plan for masks to be covered by government-sponsored health plans.

 

 

Study Limitations—Several limitations exist in our study that should be discussed. Although the data collectors observed a large number of individuals, each hospital entrance was only observed for 1 half-day morning session. There may be variations in the number of people wearing a mask at different times of day and different days of the week with fluctuations in hospital traffic. Although data were collected at a variety of hospitals representing the diverse health care delivery models available in the United States, the NYC hospitals included in this study may have different resources available for infection-prevention strategies than hospitals across the country, given NYC’s unique population density and demographics.

Study Strengths—The generalizability of the study should be recognized. Data were collected by all major health care delivery models available in the United States—private, state, city, and federal hospital systems. This study can be easily replicated in other health care delivery systems to further investigate potential gaps in mask usage and infection prevention. Repeating this study in areas where a large portion of the population does not believe in the virus also will likely show lower levels of mask use.

Conclusion

As the country grapples with vaccine hesitancy and with the new variants of SARS-CoV-2, continued universal masking is still imperative. The effectiveness of universal masking has been demonstrated, and with the combination of vaccinations, we can be assured that the world will continue to emerge from the pandemic.

References
  1. World Health Organization. Advice on the use of masks in the context of COVID-19. Interim guidance (6 April 2020). Accessed November 8, 2021. https://apps.who.int/iris/bitstream/handle/10665/331693/WHO-2019-nCov-IPC_Masks-2020.3-eng.pdf?sequence=1ceisAllowed=y
  2. World Health Organization. Advice on the use of masks in the context of COVID-19. Interim guidance (5 June 2020). Accessed November 8, 2021. https://apps.who.int/iris/bitstream/handle/10665/332293/WHO- 2019-nCov-IPC_Masks-2020.4-eng.pdf?sequence=1&isAllowed=y
  3. Fisher KA, Barile JP, Guerin RJ, et al. Factors associated with cloth face covering use among adults during the COVID-19 pandemic—United States, April and May 2020. MMWR Morb Mortal Wkly Rep. 2020;69:933-937.
  4. Centers for Disease Control and Prevention. Coronavirus disease 2019 (COVID-19). Considerations for wearing masks (19 April 2021). Accessed November 10, 2021. https://www.cdc.gov/coronavirus/2019-ncov/prevent-getting-sick/cloth-face-cover-guidance.html
  5. Conly J, Seto WH, Pittet D, et al. Use of medical face masks versus particulate respirators as a component of personal protective equipment for health care workers in the context of the COVID-19 pandemic. Antimicrob Resist Infect Control. 2020;9:126. 
  6. Chu DK, Akl EA, Duda S, et al; COVID-19 Systematic Urgent Review Group Effort (SURGE) study authors. Physical distancing, face masks, and eye protection to prevent person-to-person transmission of SARS-CoV-2 and COVID-19: a systematic review and meta-analysis. Lancet. 2020;395:1973-1987.
  7. Huang, P. Coronavirus FAQs: Why can’t the CDC make up its mind about airborne transmission? NPR. September 25, 2020. Accessed November 8, 2021. https://www.npr.org/sections/goatsandsoda/2020/09/25/916624967/coronavirus-faqs-why-cant-the-cdc-make-up-its-mind-about-airborne-transmission
  8. Centers for Disease Control and Prevention. Coronavirus disease 2019 (COVID-19). How COVID-19 spreads (14 July 2021). Accessed November 10, 2021. https://www.cdc.gov/coronavirus/2019-ncov/prevent-getting-sick/how-covid-spreads.html
  9. Wiersinga WJ, Rhodes A, Cheng AC, et al. Pathophysiology, transmission, diagnosis, and treatment of coronavirus disease 2019 (COVID-19): a review. JAMA. 2020;324:782-793. 
  10. Klompas M, Morris CA, Shenoy ES. Universal masking in the covid-19 era. N Engl J Med. 2020;383:E9.
  11. Middleton JD, Lopes H. Face masks in the covid-19 crisis: caveats, limits, and priorities. BMJ. 2020;369:m2030.
  12. Cheng KK, Lam TH, Leung CC. Wearing face masks in the community during the COVID-19 pandemic: altruism and solidarity [published online April 16, 2020]. Lancet. doi:10.1016/S0140-6736(20)30918-1
  13. Javid B, Weekes MP, Matheson NJ. Covid-19: should the public wear face masks? BMJ. 2020;369:m1442.
  14. Gandhi M, Beyrer C, Goosby E. Masks do more than protect others during COVID-19: reducing the inoculum of SARS-CoV-2 to protect the wearer. J Gen Intern Med. 2020;35:3063-3066.
  15. Ngonghala CN, Iboi EA, Gumel AB. Could masks curtail the post-lockdown resurgence of COVID-19 in the US? Math Biosci. 2020;329:108452. doi:10.1016/j.mbs.2020.108452
  16. Yi-Fong Su V, Yen YF, Yang KY, et al. Masks and medical care: two keys to Taiwan’s success in preventing COVID-19 spread. Travel Med Infect Dis. 2020;38:101780.
  17. Lim S, Yoon HI, Song KH, et al. Face masks and containment of COVID-19: experience from South Korea. J Hosp Infect. 2020;106:206-207.
  18. Fisman DN, Greer AL, Tuite AR. Bidirectional impact of imperfect mask use on reproduction number of COVID-19: a next generation matrix approach. Infect Dis Model. 2020;5:405-408.
  19. Centers for Disease Control and Prevention. COVID data tracker. United States COVID-19 cases, deaths, and laboratory testing (NAATs) by state, territory, and jurisdiction. Accessed July 6, 2021. https://covid.cdc.gov/covid-data-tracker/#cases_totalcases
  20. Francescani C. Timeline: the first 100 days of New York Gov. Andrew Cuomo’s COVID-19 response. ABC News. June 17, 2020. Accessed November 8, 2021. https://abcnews.go.com/US/News/timeline-100-days-york-gov-andrew-cuomos-covid/story?id=71292880
  21. Zhang R, Li Y, Zhang AL, et al. Identifying airborne transmission as the dominant route for the spread of COVID-19. Proc Natl Acad Sci U S A. 2020;117:14857-14863. 
  22. Verma S, Dhanak M, Frankenfield J. Visualizing the effectiveness of face masks in obstructing respiratory jets. Phys Fluids (1994). 2020;32:061708.
  23. Yang W, Shaff J, Shaman J. COVID-19 transmission dynamics and effectiveness of public health interventions in New York City during the 2020 spring pandemic wave. medRxiv. Preprint posted online September 9, 2020. doi:10.1101/2020.09.08.20190710
  24. Zavascki AP, Falci DR. Clinical characteristics of covid-19 in China. N Engl J Med. 2020;382:1859. 
  25. Zhu J, Ji P, Pang J, et al. Clinical characteristics of 3062 COVID-19 patients: a meta-analysis. J Med Virol. 2020;92:1902-1914. doi:10.1002/jmv.25884
  26. Sommerstein R, Fux CA, Vuichard-Gysin D, et al. Risk of SARS-CoV-2 transmission by aerosols, the rational use of masks, and protection of healthcare workers from COVID-19. Antimicrob Resist Infect Control. 2020;9:100.
  27. Stone TE, Kunaviktikul W, Omura M, et al. Facemasks and the covid 19 pandemic: what advice should health professionals be giving the general public about the wearing of facemasks? Nurs Health Sci. 2020;22:339-342.
  28. Tam VC, Tam SY, Poon WK, et al. A reality check on the use of face masks during the COVID-19 outbreak in Hong Kong. EClinicalMedicine. 2020;22:100356.
  29. Chen YJ, Qin G, Chen J, et al. Comparison of face-touching behaviors before and during the coronavirus disease 2019 pandemic. JAMA Netw Open. 2020;3:e2016924. 
  30. O’Dowd K, Nair KM, Forouzandeh P, et al. Face masks and respirators in the fight against the COVID-19 pandemic: a review of current materials, advances and future perspectives. Materials (Basel). 2020;13:3363.
  31. Matuschek C, Moll F, Fangerau H, et al. Face masks: benefits and risks during the COVID-19 crisis. Eur J Med Res. 2020;25:32.
  32. Chan NC, Li K, Hirsh J. Peripheral oxygen saturation in older persons wearing nonmedical face masks in community settings. JAMA. 2020;324:2323-2324. doi:10.1001/jama.2020.21905
  33. Lamb MRKandula SShaman JDifferential COVID‐19 case positivity in New York City neighborhoods: socioeconomic factors and mobilityInfluenza Other Respir Viruses2021;15:209-217. doi:10.1111/irv.12816
References
  1. World Health Organization. Advice on the use of masks in the context of COVID-19. Interim guidance (6 April 2020). Accessed November 8, 2021. https://apps.who.int/iris/bitstream/handle/10665/331693/WHO-2019-nCov-IPC_Masks-2020.3-eng.pdf?sequence=1ceisAllowed=y
  2. World Health Organization. Advice on the use of masks in the context of COVID-19. Interim guidance (5 June 2020). Accessed November 8, 2021. https://apps.who.int/iris/bitstream/handle/10665/332293/WHO- 2019-nCov-IPC_Masks-2020.4-eng.pdf?sequence=1&isAllowed=y
  3. Fisher KA, Barile JP, Guerin RJ, et al. Factors associated with cloth face covering use among adults during the COVID-19 pandemic—United States, April and May 2020. MMWR Morb Mortal Wkly Rep. 2020;69:933-937.
  4. Centers for Disease Control and Prevention. Coronavirus disease 2019 (COVID-19). Considerations for wearing masks (19 April 2021). Accessed November 10, 2021. https://www.cdc.gov/coronavirus/2019-ncov/prevent-getting-sick/cloth-face-cover-guidance.html
  5. Conly J, Seto WH, Pittet D, et al. Use of medical face masks versus particulate respirators as a component of personal protective equipment for health care workers in the context of the COVID-19 pandemic. Antimicrob Resist Infect Control. 2020;9:126. 
  6. Chu DK, Akl EA, Duda S, et al; COVID-19 Systematic Urgent Review Group Effort (SURGE) study authors. Physical distancing, face masks, and eye protection to prevent person-to-person transmission of SARS-CoV-2 and COVID-19: a systematic review and meta-analysis. Lancet. 2020;395:1973-1987.
  7. Huang, P. Coronavirus FAQs: Why can’t the CDC make up its mind about airborne transmission? NPR. September 25, 2020. Accessed November 8, 2021. https://www.npr.org/sections/goatsandsoda/2020/09/25/916624967/coronavirus-faqs-why-cant-the-cdc-make-up-its-mind-about-airborne-transmission
  8. Centers for Disease Control and Prevention. Coronavirus disease 2019 (COVID-19). How COVID-19 spreads (14 July 2021). Accessed November 10, 2021. https://www.cdc.gov/coronavirus/2019-ncov/prevent-getting-sick/how-covid-spreads.html
  9. Wiersinga WJ, Rhodes A, Cheng AC, et al. Pathophysiology, transmission, diagnosis, and treatment of coronavirus disease 2019 (COVID-19): a review. JAMA. 2020;324:782-793. 
  10. Klompas M, Morris CA, Shenoy ES. Universal masking in the covid-19 era. N Engl J Med. 2020;383:E9.
  11. Middleton JD, Lopes H. Face masks in the covid-19 crisis: caveats, limits, and priorities. BMJ. 2020;369:m2030.
  12. Cheng KK, Lam TH, Leung CC. Wearing face masks in the community during the COVID-19 pandemic: altruism and solidarity [published online April 16, 2020]. Lancet. doi:10.1016/S0140-6736(20)30918-1
  13. Javid B, Weekes MP, Matheson NJ. Covid-19: should the public wear face masks? BMJ. 2020;369:m1442.
  14. Gandhi M, Beyrer C, Goosby E. Masks do more than protect others during COVID-19: reducing the inoculum of SARS-CoV-2 to protect the wearer. J Gen Intern Med. 2020;35:3063-3066.
  15. Ngonghala CN, Iboi EA, Gumel AB. Could masks curtail the post-lockdown resurgence of COVID-19 in the US? Math Biosci. 2020;329:108452. doi:10.1016/j.mbs.2020.108452
  16. Yi-Fong Su V, Yen YF, Yang KY, et al. Masks and medical care: two keys to Taiwan’s success in preventing COVID-19 spread. Travel Med Infect Dis. 2020;38:101780.
  17. Lim S, Yoon HI, Song KH, et al. Face masks and containment of COVID-19: experience from South Korea. J Hosp Infect. 2020;106:206-207.
  18. Fisman DN, Greer AL, Tuite AR. Bidirectional impact of imperfect mask use on reproduction number of COVID-19: a next generation matrix approach. Infect Dis Model. 2020;5:405-408.
  19. Centers for Disease Control and Prevention. COVID data tracker. United States COVID-19 cases, deaths, and laboratory testing (NAATs) by state, territory, and jurisdiction. Accessed July 6, 2021. https://covid.cdc.gov/covid-data-tracker/#cases_totalcases
  20. Francescani C. Timeline: the first 100 days of New York Gov. Andrew Cuomo’s COVID-19 response. ABC News. June 17, 2020. Accessed November 8, 2021. https://abcnews.go.com/US/News/timeline-100-days-york-gov-andrew-cuomos-covid/story?id=71292880
  21. Zhang R, Li Y, Zhang AL, et al. Identifying airborne transmission as the dominant route for the spread of COVID-19. Proc Natl Acad Sci U S A. 2020;117:14857-14863. 
  22. Verma S, Dhanak M, Frankenfield J. Visualizing the effectiveness of face masks in obstructing respiratory jets. Phys Fluids (1994). 2020;32:061708.
  23. Yang W, Shaff J, Shaman J. COVID-19 transmission dynamics and effectiveness of public health interventions in New York City during the 2020 spring pandemic wave. medRxiv. Preprint posted online September 9, 2020. doi:10.1101/2020.09.08.20190710
  24. Zavascki AP, Falci DR. Clinical characteristics of covid-19 in China. N Engl J Med. 2020;382:1859. 
  25. Zhu J, Ji P, Pang J, et al. Clinical characteristics of 3062 COVID-19 patients: a meta-analysis. J Med Virol. 2020;92:1902-1914. doi:10.1002/jmv.25884
  26. Sommerstein R, Fux CA, Vuichard-Gysin D, et al. Risk of SARS-CoV-2 transmission by aerosols, the rational use of masks, and protection of healthcare workers from COVID-19. Antimicrob Resist Infect Control. 2020;9:100.
  27. Stone TE, Kunaviktikul W, Omura M, et al. Facemasks and the covid 19 pandemic: what advice should health professionals be giving the general public about the wearing of facemasks? Nurs Health Sci. 2020;22:339-342.
  28. Tam VC, Tam SY, Poon WK, et al. A reality check on the use of face masks during the COVID-19 outbreak in Hong Kong. EClinicalMedicine. 2020;22:100356.
  29. Chen YJ, Qin G, Chen J, et al. Comparison of face-touching behaviors before and during the coronavirus disease 2019 pandemic. JAMA Netw Open. 2020;3:e2016924. 
  30. O’Dowd K, Nair KM, Forouzandeh P, et al. Face masks and respirators in the fight against the COVID-19 pandemic: a review of current materials, advances and future perspectives. Materials (Basel). 2020;13:3363.
  31. Matuschek C, Moll F, Fangerau H, et al. Face masks: benefits and risks during the COVID-19 crisis. Eur J Med Res. 2020;25:32.
  32. Chan NC, Li K, Hirsh J. Peripheral oxygen saturation in older persons wearing nonmedical face masks in community settings. JAMA. 2020;324:2323-2324. doi:10.1001/jama.2020.21905
  33. Lamb MRKandula SShaman JDifferential COVID‐19 case positivity in New York City neighborhoods: socioeconomic factors and mobilityInfluenza Other Respir Viruses2021;15:209-217. doi:10.1111/irv.12816
Issue
Cutis - 108(6)
Issue
Cutis - 108(6)
Page Number
333-337
Page Number
333-337
Publications
Publications
Topics
Article Type
Display Headline
Proper Use and Compliance of Facial Masks During the COVID-19 Pandemic: An Observational Study of Hospitals in New York City
Display Headline
Proper Use and Compliance of Facial Masks During the COVID-19 Pandemic: An Observational Study of Hospitals in New York City
Sections
Inside the Article

Practice Points

  • Enormous financial and human resources have been utilized by health care systems to prevent the spread of COVID-19 in health care settings, including universal temperature checks, clinical symptom triage, and masking policies. Despite these mitigation practices, mask noncompliance continues to be a major problem in hospitals.
  • Mask compliance among 600 individuals entering 4 New York City hospitals was observed to be 78%, despite months of policies for universal masking and the city’s high mortality rates during the first COVID-19 wave.
  • Masks have been shown to reduce the spread of COVID-19, and proper mask compliance is an important issue that must be addressed by health care administrations and governmental agencies.
Disallow All Ads
Content Gating
No Gating (article Unlocked/Free)
Alternative CME
Disqus Comments
Default
Use ProPublica
Hide sidebar & use full width
render the right sidebar.
Conference Recap Checkbox
Not Conference Recap
Clinical Edge
Display the Slideshow in this Article
Medscape Article
Display survey writer
Reuters content
Disable Inline Native ads
WebMD Article
Article PDF Media

Apixaban noninferior to low-molecular-weight heparin in cancer-associated VTE

Article Type
Changed
Tue, 12/07/2021 - 12:22

Background: VTE is common in patients with cancer and can lead to serious complications and death. Relatively recently, the use of edoxaban or rivaroxaban was recommended by major guidelines for the treatment of cancer-associated VTE. Previous studies have demonstrated a higher risk of major bleeding when compared with low-molecular-weight heparin. Whether oral apixaban can be safely used in this setting is unknown.

Dr. Rex Hermansen, division of hospital medicine, Mount Sinai Health System, New York
Dr. Rex Hermansen


Study design: Randomized, controlled, open-label, noninferiority clinical trial.

Setting: Multinational study with patients enrolled in nine European countries, Israel, and the United States.

Synopsis: Adult patients with confirmed cancer who had a new diagnosis of proximal lower-limb deep vein thrombosis or pulmonary embolism were enrolled in the trial. Of those enrolled, 1,170 patients underwent randomization to receive either oral apixaban twice daily or subcutaneous dalteparin once daily. The primary outcome was recurrent deep vein thrombosis or pulmonary embolism. The principal safety outcome was major bleeding. Researchers followed patients for 7 months after randomization. The primary outcome occurred in 32 of 576 patients (5.6%) in the apixaban group and 46 of 579 patients (7.9%) in the dalteparin group (hazard ratio, 0.63; 95% CI, 0.37-1.07). Major bleeding occurred in 22 patients (3.8%) in the apixaban group and 23 patients (4.0%) in the dalteparin group (HR, 0.82; 95% CI, 0.40-1.69). Limitations were the open-label trial design; the exclusion of patients with primary brain tumors, cerebral metastases, or acute leukemia; and the sample size being powered for the primary outcome, rather than to allow definitive conclusions about bleeding. Additionally, long-term data are needed as patients were followed for only 7 months.

Bottom line: Apixaban was noninferior to subcutaneous dalteparin for the treatment of VTE in patients with cancer and did not increase bleeding.

Citation: Agnelli G et al. Apixaban for the treatment of venous thromboembolism associated with cancer. N Engl J Med. 2020 Apr 23;382:1599-607. doi: 10.1056/NEJMoa1915103.

Dr. Hermansen is a hospitalist in the Division of Hospital Medicine, Mount Sinai Health System, New York.

Publications
Topics
Sections

Background: VTE is common in patients with cancer and can lead to serious complications and death. Relatively recently, the use of edoxaban or rivaroxaban was recommended by major guidelines for the treatment of cancer-associated VTE. Previous studies have demonstrated a higher risk of major bleeding when compared with low-molecular-weight heparin. Whether oral apixaban can be safely used in this setting is unknown.

Dr. Rex Hermansen, division of hospital medicine, Mount Sinai Health System, New York
Dr. Rex Hermansen


Study design: Randomized, controlled, open-label, noninferiority clinical trial.

Setting: Multinational study with patients enrolled in nine European countries, Israel, and the United States.

Synopsis: Adult patients with confirmed cancer who had a new diagnosis of proximal lower-limb deep vein thrombosis or pulmonary embolism were enrolled in the trial. Of those enrolled, 1,170 patients underwent randomization to receive either oral apixaban twice daily or subcutaneous dalteparin once daily. The primary outcome was recurrent deep vein thrombosis or pulmonary embolism. The principal safety outcome was major bleeding. Researchers followed patients for 7 months after randomization. The primary outcome occurred in 32 of 576 patients (5.6%) in the apixaban group and 46 of 579 patients (7.9%) in the dalteparin group (hazard ratio, 0.63; 95% CI, 0.37-1.07). Major bleeding occurred in 22 patients (3.8%) in the apixaban group and 23 patients (4.0%) in the dalteparin group (HR, 0.82; 95% CI, 0.40-1.69). Limitations were the open-label trial design; the exclusion of patients with primary brain tumors, cerebral metastases, or acute leukemia; and the sample size being powered for the primary outcome, rather than to allow definitive conclusions about bleeding. Additionally, long-term data are needed as patients were followed for only 7 months.

Bottom line: Apixaban was noninferior to subcutaneous dalteparin for the treatment of VTE in patients with cancer and did not increase bleeding.

Citation: Agnelli G et al. Apixaban for the treatment of venous thromboembolism associated with cancer. N Engl J Med. 2020 Apr 23;382:1599-607. doi: 10.1056/NEJMoa1915103.

Dr. Hermansen is a hospitalist in the Division of Hospital Medicine, Mount Sinai Health System, New York.

Background: VTE is common in patients with cancer and can lead to serious complications and death. Relatively recently, the use of edoxaban or rivaroxaban was recommended by major guidelines for the treatment of cancer-associated VTE. Previous studies have demonstrated a higher risk of major bleeding when compared with low-molecular-weight heparin. Whether oral apixaban can be safely used in this setting is unknown.

Dr. Rex Hermansen, division of hospital medicine, Mount Sinai Health System, New York
Dr. Rex Hermansen


Study design: Randomized, controlled, open-label, noninferiority clinical trial.

Setting: Multinational study with patients enrolled in nine European countries, Israel, and the United States.

Synopsis: Adult patients with confirmed cancer who had a new diagnosis of proximal lower-limb deep vein thrombosis or pulmonary embolism were enrolled in the trial. Of those enrolled, 1,170 patients underwent randomization to receive either oral apixaban twice daily or subcutaneous dalteparin once daily. The primary outcome was recurrent deep vein thrombosis or pulmonary embolism. The principal safety outcome was major bleeding. Researchers followed patients for 7 months after randomization. The primary outcome occurred in 32 of 576 patients (5.6%) in the apixaban group and 46 of 579 patients (7.9%) in the dalteparin group (hazard ratio, 0.63; 95% CI, 0.37-1.07). Major bleeding occurred in 22 patients (3.8%) in the apixaban group and 23 patients (4.0%) in the dalteparin group (HR, 0.82; 95% CI, 0.40-1.69). Limitations were the open-label trial design; the exclusion of patients with primary brain tumors, cerebral metastases, or acute leukemia; and the sample size being powered for the primary outcome, rather than to allow definitive conclusions about bleeding. Additionally, long-term data are needed as patients were followed for only 7 months.

Bottom line: Apixaban was noninferior to subcutaneous dalteparin for the treatment of VTE in patients with cancer and did not increase bleeding.

Citation: Agnelli G et al. Apixaban for the treatment of venous thromboembolism associated with cancer. N Engl J Med. 2020 Apr 23;382:1599-607. doi: 10.1056/NEJMoa1915103.

Dr. Hermansen is a hospitalist in the Division of Hospital Medicine, Mount Sinai Health System, New York.

Publications
Publications
Topics
Article Type
Sections
Disallow All Ads
Content Gating
No Gating (article Unlocked/Free)
Alternative CME
Disqus Comments
Default
Use ProPublica
Hide sidebar & use full width
render the right sidebar.
Conference Recap Checkbox
Not Conference Recap
Clinical Edge
Display the Slideshow in this Article
Medscape Article
Display survey writer
Reuters content
Disable Inline Native ads
WebMD Article

Antibiotics vs. placebo in acute uncomplicated diverticulitis

Article Type
Changed
Fri, 12/03/2021 - 14:59

Background: Antibiotic therapy is considered the standard of care for acute uncomplicated diverticulitis. Over the past decade, randomized clinical trials have suggested that treatment with antibiotics may be noninferior to observation with supportive care; however, there have not been any blinded, placebo-controlled trials to provide high-quality evidence.

Dr. Ariel Y. Elyahu, division of hospital medicine, Mount Sinai Health System, New York
Dr. Ariel Y. Elyahu


Study design: Placebo-controlled, double-blinded, randomized noninferiority trial.

Setting: Four centers in New Zealand and Australia.

Synopsis: Researchers randomized 180 patients hospitalized for acute uncomplicated diverticulitis with Hinchey 1a CT findings (i.e., phlegmon without abscess) into two groups treated with either antibiotics (intravenous cefuroxime and oral metronidazole followed by oral amoxicillin/clavulanic acid) or placebo for 7 days. Median lengths of stay between the antibiotic (40.0 hours) and placebo (45.8 hours) groups were not significantly different (5.9 hours difference between groups; 95% CI, –3.7 to 15.5; P = .2). Additionally, there were no significant differences in the secondary outcomes of readmission at 7 days and 30 days or in need for procedural intervention, mortality, pain scores at 24 hours, or change in white blood cell count.

Notably, though this study was adequately powered to detect differences in length of stay, it was not powered to detect differences in clinical outcomes, including death or the need for surgery. The exclusion of patients with language barriers raises concerns regarding the generalizability of the results.

Bottom line: Antibiotic therapy does not decrease length of hospital stay when compared with placebo for patients with acute uncomplicated diverticulitis.

Citation: Jaung R et al. Antibiotics do not reduce length of hospital stay for uncomplicated diverticulitis in a pragmatic double-blind randomized trial. Clin Gastroenterol Hepatol. 2020 Mar;S1542-3565(20):30426-2. doi: 10.1016/j.cgh.2020.03.049.

Dr. Elyahu is a hospitalist in the Division of Hospital Medicine, Mount Sinai Health System, New York.

Publications
Topics
Sections

Background: Antibiotic therapy is considered the standard of care for acute uncomplicated diverticulitis. Over the past decade, randomized clinical trials have suggested that treatment with antibiotics may be noninferior to observation with supportive care; however, there have not been any blinded, placebo-controlled trials to provide high-quality evidence.

Dr. Ariel Y. Elyahu, division of hospital medicine, Mount Sinai Health System, New York
Dr. Ariel Y. Elyahu


Study design: Placebo-controlled, double-blinded, randomized noninferiority trial.

Setting: Four centers in New Zealand and Australia.

Synopsis: Researchers randomized 180 patients hospitalized for acute uncomplicated diverticulitis with Hinchey 1a CT findings (i.e., phlegmon without abscess) into two groups treated with either antibiotics (intravenous cefuroxime and oral metronidazole followed by oral amoxicillin/clavulanic acid) or placebo for 7 days. Median lengths of stay between the antibiotic (40.0 hours) and placebo (45.8 hours) groups were not significantly different (5.9 hours difference between groups; 95% CI, –3.7 to 15.5; P = .2). Additionally, there were no significant differences in the secondary outcomes of readmission at 7 days and 30 days or in need for procedural intervention, mortality, pain scores at 24 hours, or change in white blood cell count.

Notably, though this study was adequately powered to detect differences in length of stay, it was not powered to detect differences in clinical outcomes, including death or the need for surgery. The exclusion of patients with language barriers raises concerns regarding the generalizability of the results.

Bottom line: Antibiotic therapy does not decrease length of hospital stay when compared with placebo for patients with acute uncomplicated diverticulitis.

Citation: Jaung R et al. Antibiotics do not reduce length of hospital stay for uncomplicated diverticulitis in a pragmatic double-blind randomized trial. Clin Gastroenterol Hepatol. 2020 Mar;S1542-3565(20):30426-2. doi: 10.1016/j.cgh.2020.03.049.

Dr. Elyahu is a hospitalist in the Division of Hospital Medicine, Mount Sinai Health System, New York.

Background: Antibiotic therapy is considered the standard of care for acute uncomplicated diverticulitis. Over the past decade, randomized clinical trials have suggested that treatment with antibiotics may be noninferior to observation with supportive care; however, there have not been any blinded, placebo-controlled trials to provide high-quality evidence.

Dr. Ariel Y. Elyahu, division of hospital medicine, Mount Sinai Health System, New York
Dr. Ariel Y. Elyahu


Study design: Placebo-controlled, double-blinded, randomized noninferiority trial.

Setting: Four centers in New Zealand and Australia.

Synopsis: Researchers randomized 180 patients hospitalized for acute uncomplicated diverticulitis with Hinchey 1a CT findings (i.e., phlegmon without abscess) into two groups treated with either antibiotics (intravenous cefuroxime and oral metronidazole followed by oral amoxicillin/clavulanic acid) or placebo for 7 days. Median lengths of stay between the antibiotic (40.0 hours) and placebo (45.8 hours) groups were not significantly different (5.9 hours difference between groups; 95% CI, –3.7 to 15.5; P = .2). Additionally, there were no significant differences in the secondary outcomes of readmission at 7 days and 30 days or in need for procedural intervention, mortality, pain scores at 24 hours, or change in white blood cell count.

Notably, though this study was adequately powered to detect differences in length of stay, it was not powered to detect differences in clinical outcomes, including death or the need for surgery. The exclusion of patients with language barriers raises concerns regarding the generalizability of the results.

Bottom line: Antibiotic therapy does not decrease length of hospital stay when compared with placebo for patients with acute uncomplicated diverticulitis.

Citation: Jaung R et al. Antibiotics do not reduce length of hospital stay for uncomplicated diverticulitis in a pragmatic double-blind randomized trial. Clin Gastroenterol Hepatol. 2020 Mar;S1542-3565(20):30426-2. doi: 10.1016/j.cgh.2020.03.049.

Dr. Elyahu is a hospitalist in the Division of Hospital Medicine, Mount Sinai Health System, New York.

Publications
Publications
Topics
Article Type
Sections
Disallow All Ads
Content Gating
No Gating (article Unlocked/Free)
Alternative CME
Disqus Comments
Default
Use ProPublica
Hide sidebar & use full width
render the right sidebar.
Conference Recap Checkbox
Not Conference Recap
Clinical Edge
Display the Slideshow in this Article
Medscape Article
Display survey writer
Reuters content
Disable Inline Native ads
WebMD Article